Civil Procedure Q's

Réussis tes devoirs et examens dès maintenant avec Quizwiz!

For venue purposes, a business entity defendant is deemed to reside in: A. Any judicial district in which the defendant is subject to the court's personal jurisdiction B. The judicial district in which the business entity is incorporated C. The judicial district in which the business entity's owner is domiciled D. Any judicial district in which the defendant is subject to the court's personal jurisdiction with respect to the action in question

A business entity defendant is deemed to reside in any judicial district in which the defendant is subject to the court's personal jurisdiction with respect to the civil action in question. The judicial district in which the business entity is incorporated is too narrow an answer. Not all business entities are incorporated. Any judicial district in which the defendant is subject to the court's personal jurisdiction is too broad an answer; the entity must be subject to the court's personal jurisdiction with respect to the particular civil action in question. The domicile of the business entity's owner is irrelevant for venue purposes.

Following a default by a defendant, the clerk may sign and enter a judgment for the amount set forth in the plaintiff's affidavit if the defaulted defendant is not an infant or an incompetent and: A. The plaintiff's claim is for a sum certain, the default was entered because the defendant failed to appear, and the defendant's failure to appear was in bad faith B. The defendant failed to appear, and the defendant's failure to appear was in bad faith C. The plaintiff's claim is for a sum certain and the default was entered because the defendant failed to appear D. The defendant failed to appear

Following a default by a defendant, the clerk may sign and enter a judgment for the amount set forth in the plaintiff's affidavit if the defaulted defendant is not an infant or an incompetent and plaintiff's claim is for a sum certain and the default was entered because the defendant failed to appear. The defendant failing to appear is not sufficient; the claim must be for a sum certain. There is no requirement that the defendant's failure to appear be in bad faith.

Which of the following are grounds for relief from judgment for which a motion must be made within a reasonable time not to exceed one year? A. That there is newly discovered evidence that by due diligence could not have been discovered in time to move for a new trial B. That the judgment is void C. That the judgment has been satisfied D. That the judgment has been reversed

Newly discovered evidence that by due diligence could not have been discovered in time to move for a new trial is a ground upon which a party can seek relief from a judgment or order, provided the party makes a motion within a reasonable time not to exceed one year. Relief based on the fact that the judgment is void, that the judgment has been satisfied, and that the judgment has been reversed all are grounds for relief, but a motion for relief based on these things must be brought within a reasonable time not limited to a year.

Which party or parties can exercise the right of removal? A. The plaintiff, the defendant, or the court B. Only the plaintiff C. Only the defendant D. The plaintiff or the defendant

Only the defendant can exercise the right of removal. A plaintiff cannot exercise the right of removal, even on the ground that a counterclaim against him could have been brought independently in a federal court. The court cannot remove a case on its own motion.

A complaint __________ create federal question jurisdiction if it alleges federal issues only in anticipation of some defense. A. May B. Will C. Will not

A complaint will not create federal question jurisdiction if it alleges federal issues only in anticipation of some defense. The federal question must appear as part of the plaintiff's cause of action as set out in a well-pleaded complaint. It is therefore sometimes necessary to determine whether certain allegations are proper in pleading the cause of action, and whether the federal element is essential to the plaintiff's case.

Under certain circumstances, a _______ may be issued without notice to the other party. A. Summary judgment B. Preliminary injunction C. Temporary restraining order

A court may grant, in its discretion, an ex parte temporary restraining order if the moving party gives specific facts in an affidavit or in the verified complaint to establish that immediate and irreparable injury will result to the moving party before the adverse party can be heard in opposition. The other requirements are that the moving party must (i) certify in writing all efforts she made to give notice of the hearing to the adverse party and the reasons why notice should not be required; and (ii) provide some security, the amount of which is determined by the court, to pay for any costs and damages incurred by the adverse party if he was wrongfully enjoined or restrained. The United States, its officers, and its agencies are not required to give security. Preliminary injunctions and summary judgments require notice to the other party in order to be issued.

When a transfer is based on the ground that venue was improper, which law applies in the transferee court? A. The law of the transferor court B. The law of either court, whichever is more convenient C. The law of the transferee court

A transfer on the ground that the original choice of venue was improper generally results in a change of the law applicable under the Erie doctrine; i.e., the law of the state in which the transferee court sits now applies. This is in contrast to a transfer on convenience grounds, in which case the law of the transferor court continues to apply.

Once a final judgment on the merits has been rendered on a particular cause of action, the claimant is prevented from asserting the same cause of action in a later lawsuit by the doctrine of claim preclusion (res judicata). When the claimant won the earlier lawsuit, the claim is said to be: A. Barred by the prior judgment B. Either merged into or barred by the prior judgment C. Merged into the prior judgment

Although both merger and bar are used to indicate that claim preclusion (res judicata) is in effect, where the claimant won the earlier lawsuit, the claim is said to be merged into the prior judgment. Where the defendant won the earlier lawsuit, the claim is said to be barred by the prior judgment. Because the question specifically states that the claimant won the earlier suit, to say that the claim either merged into or is barred by the prior judgment is incorrect.

A car collector bought a car with gold leaf paint from a manufacturer. During the first month, all the gold leaf paint peeled off. The collector sued the manufacturer, and during discovery served an interrogatory asking the manufacturer to identify all other purchasers of the gold leaf paint over the previous 10 years. The manufacturer was aware that only about 25 of the 2 million buyers of its cars have ordered the gold leaf option. The manufacturer has retained copies of all sales forms, but has not maintained separate files of the buyers of each particular option. In a court using the federal rules, what are the manufacturer's obligations with respect to the collector's interrogatory? A. It must search its files and then disclose the information. B. It may allow the collector to search the records himself. C. It may respond by stating that only about 25 of the 2 million buyers ordered the option. D. It may state that searching the records is too great a burden and so it is excused from answering the interrogatory.

Assuming the interrogatory is otherwise proper, the manufacturer may search the 2 million order forms itself or it may allow the collector access to the files. In a situation where desired information may be ascertained from the business records of the party on whom the interrogatory was served, and where the burden of finding the information is substantially the same for the party serving the interrogatory as for the party served, it is a sufficient answer to provide the serving party reasonable opportunity to examine the records. [Fed. R. Civ. P. 33] (A) is incorrect because instead of searching the files itself, the manufacturer can allow the collector access to do it. (C) is incorrect because it is not responsive to the interrogatory. (D) is incorrect because a party is not relieved of its duty to answer just because the search is burdensome.

When a defendant attempts to remove a case from state court to federal court, the state court _______ have had subject matter jurisdiction over the case. A. Need not B. Must C. Must not

By statute, the state court need not have had subject matter jurisdiction over the case. A federal court may decide a claim in a removed civil action even if the state court had no subject matter jurisdiction. Hence, it is not correct that the state court must or must not have had subject matter jurisdiction over the case.

A party's state citizenship for purposes of diversity jurisdiction is determined: A. When the lawsuit is filed B. When the cause of action accrues C. When the lawsuit is filed, but if a party changes his state citizenship after the lawsuit is filed, the new state citizenship controls D. When the lawsuit is filed (for a claimant) or when process is served (for a defendant)

Diversity of citizenship is determined when the lawsuit is filed. Diversity need not exist when the cause of action accrues, and it is not defeated if a party changes his state citizenship after the lawsuit is filed. Hence, those choices are incorrect. State citizenship for diversity purposes is not controlled by service of process. Thus, the choice that includes "when process is served (for a defending party)" is incorrect.

For claim preclusion (res judicata) to apply, it is not necessary that the judgment be: A. Valid B. Satisfied C. On the merits D. Final

For claim preclusion (res judicata) to apply, it is not necessary that the judgment be satisfied. Before merger or bar apply, it must be shown that (i) the earlier judgment is a valid, final judgment "on the merits"; (ii) the cases are brought by the same claimant against the same defendant; and (iii) the same "cause of action" (or "claim") is involved in the later lawsuit.

For a claim brought under diversity jurisdiction, __________ is required to be alleged as damages to satisfy the jurisdictional amount A. The amount of $75,000 or more B. The amount of $100,000 or more C. An amount that exceeds $100,000 D. An amount that exceeds $75,000

For claims brought under diversity jurisdiction, an amount that exceeds $75,000 is required to be alleged as damages to satisfy the jurisdictional amount, exclusive of interest and costs. Hence, answers with an amount that exceeds $100,000 and the amount of $100,000 or more are incorrect. It is not enough for the claim to involve exactly $75,000; it must be in excess of that amount. Thus, the answer with the amount of $75,000 or more is incorrect.

In order to satisfy federal question jurisdiction, the federal question must appear in: A. The plaintiff's complaint B. Either the plaintiff's complaint or the defendant's answer C. Either the plaintiff's complaint or the defendant's answer, counterclaim, or cross-claim D. Either the plaintiff's complaint or the defendant's answer or counterclaim

In order to satisfy federal question jurisdiction, the federal question must appear in the plaintiff's complaint. The federal question must appear as part of the plaintiff's cause of action as set out in a well-pleaded complaint. Federal question jurisdiction is not satisfied if the federal question appears in the defendant's answer, counterclaim, or cross-claim.

Select the option that best completes the sentence describing the scope of discovery under the Federal Rules. Parties may obtain discovery of any _______ matter that is relevant to any party's claim or defense and proportional to the needs of the case. A. Documentary B. Admissible C. Nonprivileged

Parties may obtain discovery of any nonprivileged matter that is relevant to any party's claim or defense and proportional to the needs of the case. "Any matter" includes both documentary evidence and individuals with knowledge; thus "documentary" is not the best choice to describe the scope of discovery under the Federal Rules. As long as the information sought is reasonably calculated to lead to admissible evidence, it is not required that the information itself be admissible at trial. Therefore, "admissible" is too limiting to describe the scope of discovery under the Federal Rules.

A plaintiff, a citizen of State A, sued a defendant, a citizen of State B, in state court in State B for breach of a contract to build a house for $200,000. The defendant counterclaimed for $300,000, alleging that the plaintiff breached an earlier contract by failing to pay for a house that the defendant had built. The plaintiff files a notice of removal to federal court in State B. Can the case properly be removed to the federal court in State B? A. Yes, because all plaintiffs are of diverse citizenships from all defendants. B. Yes, unless the defendant objects to removal. C. No, unless the defendant joins in the removal. D. No, because only defendants may remove.

Removal is not proper. A plaintiff may not remove on the basis of a counterclaim against him that could have been brought in federal court. Additionally, only defendants may remove a case to federal court. For these reasons, (A) and (B) are incorrect. (C) is incorrect because removal by a plaintiff is improper even if the defendant joins in removal. An argument could be made that, because the defendant has her own right to seek removal, the court should treat her "joining" the plaintiff's removal request as her own request for removal. Here, however, the defendant has no independent right of removal because she is a citizen of the forum state.

A customer became ill from meat he purchased at the "The Corner Grocery" store in City A. The legal name of the store is "The Corner Grocery, Inc." The customer's attorney instituted a diversity action in federal district court, mistakenly naming as the defendant "The Corner Grocery" in City B, whose legal name is "The Corner Grocery Company, Ltd." The two are separate legal entities, but their stock is owned by the same persons in the same proportions. The complaint specifically described the store from which the pork was purchased, including its City A address. Approximately one month after filing the complaint, and after the statute of limitations had expired, the customer's attorney served process on an individual who is the president and 50% shareholder of both stores. The named defendant, the City B store, then filed its answer denying liability on the grounds that the customer never visited the City B store and never bought anything there. Upon learning that he had named the wrong store as defendant, the customer's attorney filed a motion to amend the complaint to substitute the name of the correct defendant for the incorrect defendant originally named. After the court allowed the amendment, the new defendant filed an answer asserting that the claim against it was barred by the statute of limitations. In response, the customer's attorney asserted that the amended complaint is not barred because it asserts a claim that arises from the same transaction as the original complaint and therefore relates back to the time of the original complaint. Is the amended complaint barred even though it relates back to the time of the original complaint? A. No, because the customer appears to have acted in good faith and exercised reasonable care in naming the proper defendant. B. No, because the new defendant received timely notice of the action so as not to be prejudiced in defending the merits and knew or should have known that the action would have been brought against it but for a mistake concerning the proper party's identity. C. Yes, because the proper defendant was not served with process within the time prescribed for service of the original complaint. D. Yes, because both the original and the amended claims are barred by the statute of limitations since service of process did not occur until after the statute of limitations expired.

The amended complaint is not barred by the statute of limitations. Under Federal Rule 15(c), an amended complaint relates back to the time of the original complaint if it asserts a claim that arises from the same transaction set forth in the original complaint and, within the time prescribed for service of process, the new defendant received such notice of the action that it will not be prejudiced in defending the merits and either knew or should have known that the action would have been brought against it but for a mistake concerning the proper party's identity. (B) is correct because it sets forth the correct criteria for changing a party or the naming of a party. (A) is incorrect because the amending party need not show reasonable care in the naming of the defendant. Rather, the defendant must find the error within the time period for serving process. (C) is incorrect. First, notice of the suit (such that the party is not prejudiced in maintaining a defense on the merits) need not be by service of process. Second, the period for gaining knowledge of the suit is not measured by the statute of limitations period, but rather by the period for service of process. (D) is incorrect for two reasons. First, the original plaintiff need not be served within the statute of limitation period. Rather, it is the filing of the lawsuit that cuts off the running of the statute of limitations. Second, as explained above, the time period for gaining knowledge of the suit is measured by the period for service of process.

Three drivers were in a traffic accident in State A. The three drivers were citizens of State A, State B, and State C, respectively. Only the driver from State C sustained any injuries and damage. The driver from State C filed a tort action against the State A driver, seeking $100,000 in damages. The State A driver believes that he was not at fault in any way and that the driver from State B was the sole cause of the accident. Assume State A does not recognize any claim of contribution among joint tortfeasors. May the State A driver assert a third-party claim against the State B driver in the pending action, alleging that the State B driver was the party at fault and should pay for the State C driver's injuries? A. Yes, because third-party claims are permitted as long as they arise from the same transaction or occurrence as the original action. B. Yes, because the State A driver's third-party claim is derivative of the State C driver's original claim. C. Yes, provided the State A driver obtains leave of the court. D. No, the State A driver may not maintain the third-party claim.

The State A driver may not maintain the third-party claim. Third-party claims may be maintained only if the defendant/third-party plaintiff alleges that the third-party defendant is liable to the defendant/third-party plaintiff for all or part of the defendant/third-party plaintiff's liability to the plaintiff. Here, the defendant/third-party plaintiff (the State A driver) can make no such claim because state law does not recognize contribution among joint tortfeasors. In other words, the State A driver may raise the State B driver's negligence as a defense, alleging that the State B driver was the sole cause of the action, but he may not implead the State B driver based on joint contribution. (A) is incorrect because the answer provides the wrong standard for third-party claims. A defending party may implead a nonparty, but only if the nonparty is or may be liable to her for any part of a judgment that the plaintiff may recover against her. This is not the case here, as discussed above. (B) is incorrect because it provides the wrong standard for a third-party impleader claim. The correct standard is stated above. (C) is also incorrect for the reasons stated above. Additionally, leave of court is required only if the third-party complaint is filed more than 14 days after service of the defendant's original answer.

A hunter residing in State A visited a website created and operated by a hunting equipment company. The company is a State B corporation, and its headquarters and all its facilities are in State B. It sells its products in several stores in State B and through its website. Its marketing efforts and most of its sales are in State A and State B. Using the company's website, the hunter ordered a hunting stand. The hunter paid for the stand and shipment to State A by providing his credit card information on the company's website. After the stand arrived at the hunter's home in State A, the hunter used the stand while on a hunting trip in State C-a very popular hunting destination. The stand collapsed, causing severe injuries to the hunter. The hunter filed a products liability action against the company in a State C court. The company filed a motion to dismiss the action on the ground that the State C court lacks personal jurisdiction over it. Although the company's website is accessible in State C, the company does not otherwise advertise there, and it does no business there. There is no evidence that the company has sold any products in State C. Does the company have sufficient contacts with State C such that a State C court may exercise personal jurisdiction over it? A. Yes, because the company's website is continuously accessible in State C so that people there may purchase products from the company at any time. B. Yes, because the company could foresee that its customers would carry the company's hunting products on hunting trips to other states-particularly states where hunting is popular. C. No, because personal jurisdiction cannot be based on Internet contacts. D. No, because the company does not have sufficient purposeful contacts with State C.

The company does not have sufficient contacts for personal jurisdiction. In addition to the state having a statute authorizing the court to exercise personal jurisdiction over a defendant, the exercise of personal jurisdiction must be constitutional, meaning that the defendant must have purposeful contacts with the forum state such that it would be fair and reasonable to exercise personal jurisdiction over him. Personal jurisdiction may be "specific," meaning that the court has personal jurisdiction over the defendant only for the plaintiff's cause of action, or it may be "general," meaning that the court has personal jurisdiction over the defendant for all causes of action. The close connection between the plaintiff's cause of action and the contact from which the case springs is usually sufficient for the court to exercise "specific" jurisdiction over the defendant. As to "general" jurisdiction, recent Supreme Court decisions require the defendant to be "at home" in the jurisdiction, a very difficult standard to meet. Here, the company had no purposeful contact with State C. It was the hunter's unilateral act that brought the stand to State C from State A. Such an act does not give the forum court personal jurisdiction over a company that places a movable object into the stream of commerce [World-Wide Volkswagen Corp. v. Woodsen]. (A) is incorrect because the mere accessibility of a company's website in a state, without more, is usually insufficient contact to support personal jurisdiction, and the claim does not arise from that contact. (B) is incorrect because the stand's transport to State C is the result of the plaintiff's unilateral activity and thus not purposeful on the part of the company. (D) is incorrect because Internet contacts can be sufficient to create personal jurisdiction in appropriate cases (e.g., when they are purposeful and the claim arises from the contacts).

An oil company incorporated in State A has its principal place of business in State B. It owns drills for oil in several eastern European countries. One of its wells in Latvia malfunctioned, causing an explosion. The explosion severely injured a number of nearby residents and damaged their homes. A number of the injured Latvians filed an action against the oil company in the United States District Court for the District of State B. Nearly all of the relevant witnesses and tangible evidence are in Latvia. If the oil company wishes to move the case away from the District of State B, which of the following grounds is likely to prove most successful? A. Dismissal for lack of subject matter jurisdiction. B. Dismissal for forum non conveniens. C. Dismissal for improper venue. D. Dismissal for lack of personal jurisdiction.

The ground most likely to be successful is forum non conveniens. Even if the court has personal jurisdiction over the defendants, subject matter over the action, and venue is proper, the court has discretion to decline to hear the case in deference to a court in another country if it determines that the other country provides a substantially more efficient, convenient, and fair forum. Here, the court might make that determination since the key evidence is in Latvia and since a trial in State B would require extensive translation. In any event, (B) represents the most likely scenario for not having trial in State B because the other choices can be eliminated. (A) is not correct because the federal court has subject matter jurisdiction over claims by citizens of other countries against citizens of U.S. states, provided the amount in controversy exceeds $75,000. Here, the oil company is a citizen of State A (the state in which it is incorporated) and State B (the state in which it has its principal place of business), and the plaintiffs are from Latvia. Due to the damage caused by the explosion and the number of severe injuries, the amount in controversy requirement is very likely satisfied. Thus, the court has subject matter jurisdiction. (C) is not correct. Federal venue in civil actions is proper in (1) the district where any defendant resides, if all defendants are residents of the state in which the district is located; and (2) the district in which a substantial part of the events or omissions giving rise to the claim occurred. A corporate defendant is deemed to reside in each district with which it has sufficient contacts to justify personal jurisdiction with respect to the action. As explained below, the oil company is subject to personal jurisdiction in State B. Thus, the oil company resides in the District of State B for venue purposes, making (C) an incorrect answer choice. (D) is not correct because the court in State B has general personal jurisdiction (that is, jurisdiction for all causes of action) over the oil company because the oil company's principal place of business is there and thus is "at home" there.

Two citizens of State A were injured in State B when the taxi in which they were riding was in an accident. The two State A citizens intend to file tort claims against the taxi driver, a citizen of State B, in federal district court seeking over $100,000 each. May or must they assert their claims in federal district court in a single civil action? A. They must assert their claims in a single action because their claims arise from the same accident. B. They may assert their claims in a single action if they wish, or they may file separate actions. C. They may not assert their claims in a single action because they are not asserting a single undivided interest. D. They may not assert their claims in a single action because each plaintiff is seeking damages for his own discrete bodily injuries.

The two plaintiffs may assert their claims in a single action. Under Federal Rule 20, parties may permissively join as plaintiffs (or be joined as defendants) whenever: (i) some claim is made by each plaintiff and against each defendant relating to or arising out of the same series of occurrences or transactions; and (ii) there is a question of fact or law common to all the parties. Here, the plaintiffs were injured in the same accident. Thus, the claims arise out the same series of occurrences and transactions, and, by necessity, the claims share common questions of law or fact (e.g., whether the acts of the taxi driver were negligent, etc.). (A) is incorrect because joinder under these facts is permissive, not mandatory. (C) is incorrect. Although plaintiffs may (and probably must under Rule 19) join together when they are asserting a single undivided interest, plaintiffs may permissively join together under the circumstances described above. (D) is incorrect. Although differences in damages may arise, the court may hear both claims together.

What is the amount in controversy requirement to establish federal question jurisdiction? A. There is no amount in controversy requirement for most federal question cases B. In excess of $75,000 C. In excess of $100,000 D. At least $75,000

There is no amount in controversy requirement in federal question cases, with the narrow exception for cases brought against defendants other than the United States, its agencies, or employees under section 23(a) of the Consumer Product Safety Act.

As a general rule, a notice of appeal must be filed with the district court within ____ days from the entry of judgment. A. 60 B. 30 C. 90 D. 28

Under Rules 3 and 4 of the Federal Rules of Appellate Procedure, a party has 30 days from the entry of judgment to file a notice of appeal with the district court. The filing period is extended to 60 days when the United States is a party to the action. 90 days and 28 days are both incorrect.

A woman sued her employer for sexual harassment. At the close of the trial, the employer made a motion for judgment as a matter of law, arguing that the woman's evidence was insufficient to establish the elements of her claim. The court denied the motion. When the jury returned a verdict in favor of the woman, the employer renewed its motion for judgment as a matter of law. In addition to the renewed motion for judgment as a matter of law, the employer also moved for a new trial, asserting that the verdict was against the weight of the evidence. The court denied both motions. If the employer appeals the denials of both the renewed motion for a judgment as a matter of law and the motion for a new trial, what is the appropriate standard of review? A. De novo for the renewed motion for judgment as a matter of law and abuse of discretion for the new trial motion. B. Clearly erroneous for the renewed motion for judgment as a matter of law and de novo for the new trial motion. C. Abuse of discretion for both. D. De novo for both.

(A) is correct. When an appellate court reviews a trial court's ruling on a post-trial motion for judgment as a matter of law (including a renewed one), it employs a de novo standard. It does so because the issue is one of law. In contrast, when an appellate court reviews a trial court's denial of a motion for new trial, it employs a more deferential standard, reversing the trial court's denial only when there is a clear showing of an abuse of discretion. Note that when a renewed motion for judgment as a matter of law and a motion for a new trial are made in the alternative and the renewed motion is granted, the court rules hypothetically on the new trial motion so that no remand is required if the ruling on the judgment as a matter of law is subsequently reversed on appeal.

An aerospace company contracted with a propeller manufacturer to provide propellers for its planes. Dissatisfied with the quality of the propellers, the aerospace company filed a diversity action against the propeller company for breach of contract, claiming breach of its express warranty and breach of the implied warranty of merchantability. At the conclusion of the trial, the aerospace company submitted proposed jury instructions listing a number of defenses that the jury could not properly consider, including the aerospace company's assumption of risk and its failure to test the propellers. The court refused to give these instructions, and instead instructed the jury on the elements of breach of express and implied warranty. The aerospace company did not object to the instructions given by the court. After the jury returned a verdict in favor of the propeller manufacturer, the aerospace company moved for a new trial on the ground that the court erroneously refused to accept the aerospace company's requested jury instructions. The court denied the motion and the aerospace company filed a timely appeal. Which statement best describes the aerospace company's rights on appeal? A. The aerospace company waived its right to object to the jury instructions. B. The appeals court may review the court's refusal to include the instructions, applying a de novo standard of review. C. The appeals court may review the court's refusal to include the instructions, applying a plain error standard of review. D. The appeals court may review the court's refusal to include the instructions, applying a harmless error standard of review.

(C) is correct. Because the aerospace company did not timely object to the instructions, the court will review them using a plain error standard of review. In order to reserve its right to appeal an error in an instruction given or, as is the case here, a failure to give an instruction, a party must object on the record before the instructions are given. If adequately preserved, the instructions are reviewed using an abuse of discretion standard. However, if the objection is not made (and therefore not preserved for full appellate review), then the court's review is limited to considering whether there was a plain error in the instruction that affected substantial rights. Thus (C) is correct, and (A), (B), and (D) are incorrect.

A company manufactured and sold a product called "True Glue." An inventor brought an action in federal court against the company, alleging that the product infringed a patent owned by him. The company denied the allegations of the inventor's complaint. In addition, it asserted a counterclaim against the inventor for breach of contract, based on a transaction between them unrelated to the inventor's claim. After the completion of discovery, the company moved for summary judgment dismissing the inventor's claim. The court granted the motion, thus leaving only the company's counterclaim for breach of contract to be adjudicated. Which of the following statements most accurately describes the inventor's right to appeal the court's grant of the motion for summary judgment? A. The inventor may immediately appeal the judgment because summary judgment is a final judgment. B. The inventor may immediately appeal the judgment because his claim does not arise out of the same transaction or occurrence as the company's claim. C. The inventor may not appeal the judgment until the company's claim against him has been adjudicated. D. The inventor may not immediately appeal the judgment unless the court provides that it is a final judgment and expressly determines that there is no just reason for delay.

(D) is correct. When a court grants summary judgment on some but not all of the claims in an action, the court's order is not final and thus not appealable unless the court expressly determines that there is no just reason to delay entry of judgment. Unless the trial judge makes that express determination, its order determining the merits of fewer than all of the claims is not a final judgment and is not appealable. For these reasons, (A) is incorrect. As discussed, summary judgment is not a final judgment unless it disposes of all of the parties' claims or the court expressly determines that there is no just reason for delay. (B) is incorrect because the grant of summary judgment dismissing a claim is not appealable simply because the remaining claim arises from a different transaction or occurrence. (C) is incorrect because the order dismissing the inventor's claim would have been appealable had the court determined there was no just reason to delay.

In federal court, and without a court order, which of the following is an authorized way to serve process on an individual? A. E-mailing a copy of the summons and complaint to the defendant B. Mailing a copy of the summons and complaint to the defendant C. Following the state rules for service of process D. Posting the summons and complaint on the defendant's property

A federal court may follow the state rules for service of process. In federal court, service by posting the summons and complaint on the defendant's property and e-mailing a copy of the summons and complaint to the defendant are not authorized methods of service. However, note that such methods would be permissible if allowed by state law. Mailing a copy of the summons and complaint to the defendant is not, by itself, an authorized method of service. Rather, in federal court, the mailing is a request to waive service, and must include a waiver form describing the impact of waiving (or not waiving) service. Of course, like above, if state rules authorize service by mail, a plaintiff can use mail for service of process.

During a trial, a motion for judgment as a matter of law ("JMOL") may be filed: A. At any time before a verdict is entered B. No later than 28 days after the entry of judgment C. At any time before submission of the case to the jury

During a trial, a motion for judgment as a matter of law ("JMOL") may be filed at any time before submission of the case to the jury. The moving party must specify in its motion the judgment sought and the law and facts on which the party is entitled to judgment. Note, however, that the nonmoving party must have been heard on the issue. The answer "at any time before a verdict is entered" is incorrect because the motion is unavailable once the case is submitted to the jury. The answer "no later than 28 days after the entry of judgment" is incorrect because the motion is unavailable once the case is submitted to the jury. It should be noted that this is the filing timeframe for a renewed motion for judgment as a matter of law ("JNOV").

During discovery, electronically stored information need not be produced if the responding party identifies it as: A. Not easily accessible B. Not stored in the form requested C. Not accessible without incurring additional cost D. Not reasonably accessible because of undue burden or cost

During discovery, electronically stored information need not be produced if the responding party identifies it as from a source that is not reasonably accessible because of undue burden or cost. "Not easily accessible" is the incorrect standard, as is not accessible without incurring additional cost. The additional cost must be undue. The fact that the information is not stored in the form requested is not a valid excuse for failure to produce electronic data. A responding party must use the form requested by the requesting party unless the responding party makes an objection that is determined to be valid by the court. Even if the objection is valid, it merely affects the form in which the information is to be produced, not the production itself.

For purposes of diversity jurisdiction, the state citizenship of an individual is determined by: A. The state in which the person has his permanent home and to which he intends to return B. The state in which the person is currently residing, even if temporarily C. The state in which the person votes D. The state in which the person has a valid driver's license or state identification card

For diversity purposes, the state citizenship of an individual is the state in which he has his permanent home and to which he intends to return. The state in which the person is currently residing, even if temporarily, is not the person's home state for diversity purposes. Although voting and holding some sort of state identification will be considered as evidence of an individual's true permanent home, the state in which the person votes and the state in which the person has a valid driver's license or state identification card are not generally determinative of the person's state citizenship.

For purposes of diversity jurisdiction, a corporation is considered to be a citizen of: A. Every state in which it is incorporated and the one state in which it has its principal place of business B. The first state in which it was incorporated and every state in which it does substantial business C. The first state in which it was incorporated and the one state in which it has its principal place of business D. Every state in which it is incorporated and every state in which it does substantial business

For purposes of diversity jurisdiction, a corporation is deemed to be a citizen of every state in which it is incorporated and the one state in which it has its principal place of business. Thus, it is possible for a corporation to have two or more state citizenships for diversity purposes. The rule for a corporation is that, in addition to its states of incorporation, a corporation is a citizen of the one state in which it has its principal place of business. It is not a citizen of every state in which it does substantial business. Likewise, in addition to the principal place of business, a corporation is deemed to be the citizen of every state in which it is incorporated, not just the first state in which it was incorporated. Thus, the choices incorporating those standards are incorrect statements of a corporation's citizenship for diversity purposes.

Which of the following statements regarding the parties involved provides a sufficient basis for applying claim preclusion? A. The earlier and the latter causes of action involved the same parties B. The earlier and the latter causes of action were brought by the same claimant against the same defendant C. The earlier and the latter causes of action were brought against the same defendant

For res judicata to apply, the earlier and latter causes of action must be brought by the same claimant against the same defendant. It is not sufficient simply that both cases involve the same parties. For example, if the defendant in the first case is the claimant in the second case, claim preclusion will not apply even though both cases involve the same parties. Similarly, it is not sufficient that both cases were brought against the same defendant. Claim preclusion will not apply unless the claimant is the same in both cases.

Absent a showing of substantial need and undue hardship, the "work product" (a document or tangible thing) made by a party or representative of a party (such as the party's attorney) is not discoverable: A. Under any circumstances B. If relevant to the litigation C. If made in anticipation of litigation

Generally, the work product-a document or tangible thing-made by a party or representative of a party (such as the party's attorney) is not discoverable if made in anticipation of litigation, unless the party requesting discovery can show substantial need and undue hardship if disclosure is not ordered. It is not the case that work product is not discoverable under any circumstances. Work product that was not made in anticipation of the litigation is obtainable. Also, work product that is relevant to the litigation may be discoverable if the work product was not made in anticipation of the litigation (or if it is not a document or tangible thing).

An elderly woman entered into a contract with a company in the business of providing home care services. Believing that she had been duped by representatives of the company, the woman commenced an action in federal court, properly based on diversity, seeking rescission of the contract. The company answered, denying the principal allegations of the woman's complaint and asserting a counterclaim against the woman for breach of contract. In addition, the company timely served a demand for a jury trial. The woman did not. Which statement best describes the roles of the judge and jury as finders of fact in the trial of the parties' claims? A. The judge will first determine the issues relating to the woman's claim for rescission, and if it concludes the rescission is not warranted, the jury will determine the issues relating to the company's breach-of-contract counterclaim. B. The jury will first determine the issues relating to the breach of contract claim, and the judge will determine the issues relating to the rescission claim that have not already been resolved by the jury. C. The jury will act as the sole finder of fact. D. The judge may either act as the sole finder of fact on both the claim and the counterclaim, or allow an advisory jury to try the issues relating to the company's counterclaim.

If legal and equitable claims are joined in one action involving common fact issues, the legal claim is tried first before the jury, and then the equitable claim is tried to the court. The jury's finding on fact issues will bind the court in the equitable claim. (A) is incorrect because all fact issues relating to the company's legal claim must first be determined by the jury. Only then may the judge try the woman's equitable claim. (C) and (D) are both incorrect because neither the jury nor judge acts as the sole finder of fact when the case has legal claims and equitable claims, and a jury has been demanded on the legal claims.

If no federal question is involved and diversity does not exist when a case is commenced, removal will: A. Be permitted if the nondiverse parties are thereafter dismissed from the action and the requirements for diversity jurisdiction are then present B. Be permitted if the nondiverse parties are thereafter dismissed from the action, the requirements for diversity jurisdiction are then present, and not more than one year has passed since the case was commenced in state court C. Not be permitted under any circumstances D. Be permitted, because subject matter jurisdiction is not required for removal

If no federal question is involved and diversity does not exist because a party is a co-citizen of an opposing party (but the amount in controversy is satisfied), removal will be permitted if the nondiverse parties are thereafter dismissed from the action, the requirements for diversity jurisdiction are then present, and not more than one year has passed since the case was commenced in state court. This rule is subject to certain limitations. Removal will not be permitted in all cases; i.e., subject matter jurisdiction is required for removal.

A corporation operated several factories that were emitting toxic chemicals into the air. The Environmental Protection Agency ("EPA") sued the corporation in federal court for violation of several environmental laws, and the jury found in favor of the EPA. One of the corporation's factories was located near a town, and some residents believed that their health had been harmed by the factory's emissions. One resident sued the corporation in federal district court, alleging damages stemming from the factory's violation of the environmental laws. The resident asserts issue preclusion (collateral estoppel) to establish the factory's violation of the environmental laws. If the court permits the resident to use issue preclusion to establish the violation, what is the likely reason? A. A nonparty in the first case is always allowed to use issue preclusion (collateral estoppel) offensively or defensively against someone who was a party in the first case. B. There was a valid, final judgment on the merits in the first case, and the same cause of action is involved in the resident's lawsuit. C. The court determined that it is fair and equitable to allow the resident to use issue preclusion (collateral estoppel) offensively. D. A party can use issue preclusion (collateral estoppel) offensively when the defendant has been found to have violated a federal law.

If the court permits the resident to use issue preclusion (collateral estoppel) to establish the violation, it is likely because the court determined that it is fair and equitable for the resident to do so. Under the traditional mutuality rule, only someone who was a party in the previous case can use issue preclusion. However, this rule has been modified to allow nonparties to use issue preclusion in certain circumstances. When a nonparty wants to use a previous judgment offensively, the court must consider whether it would be fair and equitable to allow the nonparty to do so. (A) is therefore incorrect because nonparties are not always allowed to use issue preclusion offensively or defensively. (B) is incorrect because the same cause of action need not be involved. The same cause of action is a requirement for claim preclusion (res judicata). (D) is an incorrect statement of the law.

In a diversity case, federal trial courts are required to apply a _______ when considering a motion for a new trial based on the excessiveness of the verdict. A. Federal law standard B. State or federal law standard, in the court's discretion C. State law standard

In a diversity case, federal trial courts are required to apply a state law standard when considering a motion for a new trial based on the excessiveness of the verdict. Federal trial courts may neither use a federal law standard, nor choose a state or federal law standard in the court's discretion.

The plaintiff corporation is incorporated in State A and has its principal place of business in State B. The defendant corporation is incorporated and has its principal place of business in State C. It also has branch offices and does substantial business in all 50 states. The cause of action is based on a breach of contract that was to be performed in State D, which has a long arm statute authorizing jurisdiction in such cases. In which of these states may the plaintiff corporation bring its action? A. In State C only, because that is the only state in which the defendant corporation is at home. B. In State D only, because that is where the cause of action arose and is thus the only proper venue. C. In States A or B only, because those are the only two states in which the plaintiff corporation resides for venue purposes. D. In States C or D only, because only those two states have personal jurisdiction and proper venue.

In a diversity case, the federal court is required to analyze the personal jurisdiction question as if it were a state court sitting in the particular state. States generally allow for general jurisdiction (i.e., jurisdiction over all causes of action) over persons and entities domiciled within their borders. Although it used to be that transacting a substantial amount of business in a state subjected a defendant to general jurisdiction in that state, the Supreme Court has narrowed this concept by requiring the defendant to be "at home" in the state. To be "at home" in a state, a defendant corporation must be incorporated in the state or have its principal place of business in the state. Thus, a court of State C would have general jurisdiction over the defendant corporation. Venue would also be proper in State C because the defendant corporation is subject to personal jurisdiction in State C with respect to the action. For specific jurisdiction (i.e., jurisdiction for the instant cause of action only), it must be determined whether the state has a long arm statute authorizing jurisdiction and whether the statute is constitutional (i.e., whether the defendant has sufficient minimum contacts with the jurisdiction such that the exercise of jurisdiction over him would be fair and reasonable). Here, State D has an applicable long arm statute that authorizes jurisdiction in breach of contract cases where the contract called for performance in the state. This type of provision is an uncontroversial provision that routinely has been held to be constitutional. Thus, personal jurisdiction also exists in State D, and venue would be proper both because a substantial part of the cause of action arose in State D and because the defendant corporation resides therein for venue purposes (because it is subject to personal jurisdiction there with respect to the action). (A) is incorrect because, although the defendant corporation is subject to general jurisdiction in State C, it is also subject to specific jurisdiction in State D, as explained above. (B) is incorrect because venue also may be based on the defendant's residence for venue purposes, as explained above. (C) is incorrect because venue is based on where the events giving rise to the cause of action took place or where the defendant (not the plaintiff) resides for venue purposes.

In a federal question case, the action is always deemed commenced for statute of limitations purposes when: A. Either the complaint is filed or process is served on the opposing party, depending on the law of the state in which the federal court sits B. Process is served on the opposing party C. The complaint is filed with the court D. The complaint is filed, if service of process occurs within a specified timeframe

In a federal question case, the action is deemed commenced for statute of limitations purposes when the complaint is filed with the court. In diversity cases (not federal question cases), the Supreme Court has held that the state rule for determining when the action is commenced applies. Thus, depending on the state rule, an action may be commenced for statute of limitations purposes when process is served on the opposing party. However, the federal rule (as described above) applies in federal question cases. The choice providing commencement when either the complaint is filed or process is served on the opposing party, depending on the law of the state in which the federal court sits is incorrect; this essentially states the rule when subject matter jurisdiction is based on diversity of citizenship. The choice providing for commencement when the complaint is filed, if service of process occurs within a specified timeframe is incorrect. Some states have such dual provisions, in that the date that the complaint is filed is used when service occurs within a certain timeframe after filing, but if that timeframe for service is exceeded, the date of service is used. But that is not the rule in federal question cases.

Select the statement that best describes the relationship between removal and venue: A. In a properly removed case, venue is proper in the federal court of the state where the case was pending, even if venue would have been improper had the plaintiff originally filed the action in the federal district court of that state B. In a properly removed case, venue is proper in the federal court of the state where the case was pending, but only if venue would have been proper had the plaintiff originally filed the action in the federal district court of that state C. Venue and removal have no correlation

In a properly removed case, venue is proper in the federal court of the state where the case was pending, even if venue would have been improper had the plaintiff originally filed the action in the federal district court of that state. This is because venue for an action removed under section 1441(a) lies in the federal district court "embracing the place where such [state] action is pending." Hence, it is not correct to state that venue and removal have no correlation.

When a plaintiff has both federal and state-based claims against a defendant and diversity jurisdiction does not exist, the federal court has: A. Discretion to exercise supplemental (pendent) jurisdiction over the state law claim if the two claims derive from a common nucleus of operative fact and are such that a plaintiff would ordinarily be expected to try them all in one judicial proceeding B. Discretion to exercise supplemental (pendent) jurisdiction over the state law claim, regardless of whether the two claims derive from a common nucleus of operative fact C. No discretion to exercise supplemental (pendent) jurisdiction over the state law claim; it must transfer all claims to state court D. No discretion to exercise supplemental (pendent) jurisdiction over the state law claim; it must do so

In some cases, the plaintiff will have both federal and state claims against the defendant. Although there may be no diversity, the federal court has discretion to exercise supplemental (pendent) jurisdiction over the claim based on state law if the two claims are so related that they are part of the same case or controversy, which essentially means that they derive from a common nucleus of operative fact and are such that a plaintiff would ordinarily be expected to try them all in one judicial proceeding. A federal court does not have discretion to exercise jurisdiction if the claims are unrelated. Hence, it is not correct that the court has discretion regardless of whether the claims are related. The court's exercise of supplemental jurisdiction in such a scenario is discretionary. Thus, both answer choices asserting that the court has no discretion to exercise supplemental (pendent) jurisdiction over the state law claim are incorrect. It need not exercise discretion, and it need not transfer the claims to state court.

The general rule is that only final orders may be appealed. However, certain interlocutory orders may be appealed as of right. Which of the following orders is immediately reviewable on appeal even if it is not a final order? A. An order dismissing defendant's counterclaims B. An order regarding copyright claims C. An order granting an injunction

Interlocutory (i.e., nonfinal) orders that may be immediately appealed include: (i) an order granting an injunction (or continuing, modifying, dissolving, or refusing to dissolve an injunction), (ii) an order appointing a receiver, or refusing to wind up or take steps to accomplish purposes of receiverships, (iii) decrees in admiralty cases that find liability but leave damages to be assessed later, (iv) a patent infringement order where only an accounting is wanting, and (v) an order whereby possession of property is changed or affected, such as orders dissolving writs of attachment and the like. An order dismissing defendant's counterclaims would not typically be subject to an interlocutory appeal absent other circumstances. Although patent infringement orders may sometimes be subject to interlocutory appeal as stated above, there is no such exception for an order regarding copyright claims.

A researcher and an analyst had employment contracts with a corporation. The corporation fired them because it believed they had conspired to give its trade secrets to a competitor. Knowing that the conspiracy allegations were not true, the researcher and the analyst sued the corporation in federal district court for breach of their employment contracts. The researcher seeks $100,000 in damages and the analyst seeks $50,000. They are both citizens of State A. The corporation is incorporated in State B, and its principal place of business is in State B. Does the federal district court have subject matter jurisdiction over both of the claims? A. Yes, the court has diversity of citizenship jurisdiction over both claims because the amounts of their related claims may be aggregated to satisfy the amount in controversy requirement. B. Yes, the court has diversity of citizenship jurisdiction over the researcher's $100,000 claim and supplemental jurisdiction over the analyst's $50,000 claim. C. No, the court does not have subject matter jurisdiction over the analyst's $50,000 claim because the claims of both plaintiffs may not be aggregated to satisfy the amount in controversy requirement. D. No, the court does not have subject matter jurisdiction over either claim because the breach of contract claim is based on state law.

The court has jurisdiction over both claims but under different bases. Generally, every claim in federal court must have a basis for federal subject matter jurisdiction. There are three main flavors of federal subject matter jurisdiction-diversity of citizenship jurisdiction, federal question jurisdiction, and supplemental jurisdiction. Diversity of citizenship jurisdiction is available when (i) there is complete diversity of citizenship, meaning that each plaintiff is a citizen of a different state from every defendant; and (ii) the amount in controversy exceeds $75,000. A natural person's citizenship is the state that is the person's permanent home. A corporation is a citizen of every state in which it is incorporated and the one state in which it has its principal place of business. The amount in controversy is determined by the plaintiff's good faith allegation. In the instant case, the facts state that the researcher and the analyst are citizens of State A, and that the corporation was incorporated and has its principal place of business in State B. As a result, complete diversity exists. The researcher claims $100,000, presumably in good faith. Therefore, the researcher may claim diversity jurisdiction to have his claim heard in federal court. The analyst, on the other hand, claims only $50,000 in damages; thus, he may not invoke diversity jurisdiction to have his claim heard in federal court. However, the analyst might invoke supplemental jurisdiction to have his claim heard in federal court. When the federal court has subject matter jurisdiction over one claim, it has discretion to exercise supplemental jurisdiction over related claims that derive from the same common nucleus of fact and are such that a plaintiff would ordinarily be expected to try them in a single judicial proceeding. Here, the analyst's claim is derived from the corporation's wrongful termination of both plaintiffs based on an allegation that they conspired to sell a trade secret to a competitor. As a result, the analyst's claim springs from the same common nucleus of fact as the researcher's claim. There are several restrictions on the use of supplemental jurisdiction when joined with a claim based solely on diversity; however, a co-plaintiff with a below-limit claim generally can use supplemental jurisdiction to have his claim heard in federal court so long as the co-plaintiff's presence in the suit does not destroy complete diversity. Here, the analyst's presence does not destroy complete diversity. Thus, the analyst may invoke supplemental jurisdiction to have his claim heard in federal court, making (B) correct. (A) is incorrect because multiple plaintiffs may not aggregate their claims against a single defendant unless they are seeking to enforce a joint right in which they have an undivided interest. Such a case is not presented under these facts. (C) is incorrect. Although it is true that the claims of both plaintiffs may not be aggregated, the answer ignores the possibility of there being supplemental jurisdiction. (D) is incorrect because state law claims may be heard in federal court under the court's diversity jurisdiction.

Which of the following factors is not required in order for an otherwise nonappealable interlocutory order to be appealable under the Interlocutory Appeals Act? A. The controlling question of law is one as to which there is a substantial ground for a difference of opinion B. The amount in controversy exceeds $75,000 C. At least two appellate court judges agree to allow an appeal D. An immediate appeal of the order may materially advance the ultimate termination of the litigation

The amount in controversy exceeds $75,000 is the correct answer because it is not a requirement for an immediate appeal under the Interlocutory Appeals Act. Review under the Interlocutory Appeals Act is discretionary with the court and may be available when: (i) the trial judge certifies that the order involves a controlling question of law as to which there is substantial ground for a difference of opinion and an appeal would materially advance the conclusion of the case, and (ii) at least two appellate court judges agree to hear the appeal.

An investor filed a civil action against her broker in a State A state court seeking $30,000 for violation of federal securities statutes. The investor is a citizen of State B and the broker is a citizen of State A. May the broker remove the action to federal district court? A. No, because the amount in controversy is too small for federal subject matter jurisdiction. B. No, because the defendant is a citizen of the state from whose court the action is removed. C. Yes, the broker may remove the action to federal district court on the basis of federal question jurisdiction. D. Yes, the broker may remove the action to federal district court on the basis of federal diversity of citizenship jurisdiction.

The broker may remove the action on the basis of federal question jurisdiction. A defendant may remove an action that could have originally been brought in the federal courts, either on the basis of a federal question being presented or on the basis of diversity of citizenship. Federal question jurisdiction is available when the plaintiff, in his well-pleaded complaint, alleges a claim that arises under federal law. In the instant case, the cause of action is based on a violation of federal securities law. Thus, a federal question has been presented, making the case removable on the basis of federal question jurisdiction. (A) is incorrect. Although diversity of citizenship jurisdiction has an amount in controversy requirement of more than $75,000, there is no amount in controversy requirement for federal question jurisdiction. (B) is also incorrect. When jurisdiction of the federal court is based solely on diversity, and one of the defendants is a citizen of the state in which the action was brought, the action is not removable. There is no similar restriction on removal when removal is based on federal question jurisdiction. (D) is incorrect because the amount in controversy ($30,000) is not sufficient to invoke diversity jurisdiction (i.e., it does not exceed $75,000).

A car buyer, a resident of State A, wished to purchase a new car. After shopping both online and in person for the best price, the buyer decided to purchase a car from a dealer in State B. The buyer drove to the State B dealership, signed a sales contract, paid for the car, and drove the car back home to State A. Three months later, the buyer was in an accident in State A while driving the new car. The other driver in the accident was a resident of State B who was just passing through State A. The other driver filed a negligence action against the buyer in a court in State B. Does the buyer have sufficient contacts with State B such that a State B court could exercise personal jurisdiction over the buyer for the negligence action? A. Yes, because the buyer had purposeful contacts with State B. B. No, because the other driver's claim does not arise from the buyer's contacts with State B, and the contacts are insufficient to create general personal jurisdiction. C. No, because the buyer's contacts with State B were not purposeful. D. No, because the buyer did not benefit in any way from the contacts with State B.

The buyer does not have sufficient contacts. In addition to the state having a statute authorizing the court to exercise personal jurisdiction over a defendant, the exercise of personal jurisdiction must be constitutional, meaning that the defendant must have purposeful contacts with the forum state such that it would be fair and reasonable to exercise personal jurisdiction over him. Personal jurisdiction may be "specific," meaning that the court has personal jurisdiction over the defendant for the complained-of cause of action, or it may be "general," meaning that the court has personal jurisdiction over the defendant for all causes of action. The close connection between the plaintiff's cause of action and the contact from which the case sprung is usually sufficient for the court to exercise "specific" jurisdiction over the defendant. As to "general" jurisdiction, recent Supreme Court decisions require the defendant to be "at home" in the jurisdiction, a very difficult standard to meet. The phrase "at home" requires the defendant to be essentially domiciled in the state. Here, the buyer did have purposeful contacts with State B, and the buyer benefited from those contacts in purchasing the car. However, the contacts were not related to the claim asserted. As a result, there is no close connection between the case and the buyer's contacts, and it would not be fair and reasonable to exercise specific jurisdiction in this instance. Furthermore, the buyer is not domiciled in State B, so he is not "at home" there. (A) is incorrect because, although the fact that the contacts are purposeful is critical to the determination whether a court has personal jurisdiction over the defendant, the purposeful contacts must be related to the plaintiff's cause of action. (C) and (D) are incorrect factual statements-the buyer did benefit from the transaction, and buying a car in State B is a purposeful activity. Note that benefit-type language is used when discussing whether the defendant's contacts were purposeful-i.e., the defendant must have purposeful contacts with the forum such that it can be said that the defendant invoked the benefits and protections of state law with respect to the transaction or incident (e.g., if the car dealer had become involved in a contract-type dispute with the buyer over the sale of the car).

A homeowner is suing a contractor in federal court for fraud and misrepresentation. Subject matter jurisdiction is based on diversity, and is clearly appropriate. The homeowner presented three witnesses, while the contractor presented one witness. The contractor's witness is impressive on the stand, and all observers believe that the contractor will prevail. The case then goes to the jury without any motions being made by either party, and the jury returns a verdict for the homeowner. If the contractor moves for a renewed motion for judgment as a matter of law, will the contractor be successful? A. Yes, if the court also believes the contractor's witness was the more credible witness. B. Yes, if the homeowner cannot reopen her case to present more evidence. C. No, because the contractor did not meet the procedural requirements. D. No, because a jury verdict based on witness testimony cannot be overturned.

The contractor will not be successful in moving for a renewed motion for judgment as a matter of law because he did not meet the procedural requirements for doing so. In most courts, a party may not move for judgment notwithstanding the verdict (JNOV) unless he has previously moved for a directed verdict at some time during the trial. In federal court terminology, a party may not make a renewed motion for judgment as a matter of law unless he has previously moved for judgment as a matter of law at some point during the trial on the issue. [Fed. R. Civ. P. 50] Hence, (A) and (B) are incorrect. (D) is incorrect because there is no such rule.

A citizen of State A filed a complaint alleging negligence by two parties, both of whom reside in State B. The complaint was filed in the United States District Court of State B. However, the accident took place in State D, the plaintiff was treated by an emergency room physician in State D, and all of the witnesses reside in State D. Two months after they filed their respective answers, which did not address any problems with personal jurisdiction or venue, the defendants filed a joint motion to transfer the case to the District of State D. May the court grant the motion? A. No, because the defendants have waived any issue as to venue by not raising the issue of venue in the answer. B. No, because a plaintiff is entitled to choose venue. C. Yes, because the defendants raised the issue in their first motion before the court. D. Yes, because transferring the case to the District of State D could be "in the interests of justice."

The court could grant the motion. Pursuant to Rule 12(b), improper venue must be raised in a defendant's first response-either in its timely motion to dismiss before the answer or in the answer, whichever is first. Here, though, venue was initially proper because the defendants resided in State B. Rather, the issue is whether the court may transfer the case from one proper venue to another. Such a transfer has no strict time limit. Rather, transfer is left to the discretion of the trial judge, and the judge may refuse transfer where the case has been pending for some time and would work a prejudice to one of the parties. Two months is likely not a sufficient period to prevent transfer of the case. Thus, (D) is correct. (C) is incorrect. First, it misstates Rule 12. Rule 12 defenses must be raised in the first motion or answer, whichever comes first. Second, transfer where venue is initially proper does not fall under the Rule 12 timing requirements. (B) is incorrect because, although a plaintiff is entitled to choose the initial venue, venue may be transferred, in the interests of justice, for the convenience of the parties and witnesses despite the plaintiff's initial choice of the forum. (A) is incorrect because, as explained, transfer of venue in the interests of justice does not fall under the "first pleading or motion" rule.

While working on a site in State A, a State B construction worker was standing near a steel crane when the crane's boom swung near a high tension power line. The worker was electrocuted and severely injured. The worker filed an action in federal district court against the power company that owns the power lines. The action seeks $500,000 and alleges that the power company's negligent construction, maintenance, and operation of the power lines caused the injury. The power company is a State A corporation and all its operations are in State A. The power company filed a third-party complaint against the owner-operator of the crane, a State B citizen. The third-party claim is based on state law and alleges that the crane's owner-operator is liable to the power company for any liability the power company has to the injured worker. The worker amended his complaint to add a state law negligence claim for $500,000 against the crane's owner-operator. Does the federal court have subject matter jurisdiction over the worker's claim against the owner-operator of the crane? A. No, because the court does not have supplemental jurisdiction over the worker's claim against the owner-operator of the crane. B. Yes, because the claim arose from the same transaction or occurrence as the worker's claim against the power company. C. Yes, because all claims asserted arose from a common nucleus of operative fact. D. Yes, because the State B worker has sufficient contacts with State A.

The court does not have subject matter jurisdiction over the worker's claim against the owner-operator of the crane. Diversity of citizenship jurisdiction is available when (i) there is complete diversity of citizenship, meaning that each plaintiff must be a citizen of a different state from every defendant; and (ii) the amount in controversy exceeds $75,000. The citizenship of an individual is his domicile, and a corporation is a citizen of every state in which it is incorporated and the one state in which it has its principal place of business. Here, the worker is from State B and the power company is a State A corporation with all its operations (and therefore its principal place of business) in State A. The plaintiff's claim is for $500,000, satisfying the amount in controversy requirement. Accordingly, there is diversity of citizenship jurisdiction over this claim. Once one claim satisfies the requirements for original federal subject matter jurisdiction, the court has discretion to exercise supplemental jurisdiction over related claims that derive from the same common nucleus of fact and are such that a plaintiff would ordinarily be expected to try them in a single judicial proceeding. However, for cases based solely on diversity, supplemental jurisdiction is not available for claims by the plaintiff against persons made parties under the impleader rules when use of supplemental jurisdiction would be contrary to the requirements of diversity jurisdiction. In the instant case, the owner-operator shares state citizenship with the worker and was made a party when the power company impleaded him on a claim for indemnity. Because a claim by the worker against the owner-operator would circumvent the complete diversity requirement, supplemental jurisdiction is not available for that claim. Hence, (A) is correct. (B) and (C) are incorrect because supplemental jurisdiction cannot be used to override the requirements of diversity jurisdiction for claims by a plaintiff against an impleaded party (as is the case in this question). (D) is an incorrect statement of law, and irrelevant. First, subject matter jurisdiction (the power to hear a case) must be distinguished from personal jurisdiction (the power over a particular defendant). A court technically may have subject matter jurisdiction (for example, when diversity jurisdiction is satisfied) without having personal jurisdiction over one of the defendants because the defendant has insufficient contacts with the jurisdiction. Furthermore, a plaintiff consents to the personal jurisdiction of the court by filing suit. Here, that means the State B worker submitted to personal jurisdiction in State A by filing suit.

A building materials supplier, who is a citizen of State A, and a contractor, who is a citizen of State B, had a contract dispute. The contractor drove to the supplier's home in State A to discuss possible settlement of the dispute. The discussions were not successful. As the contractor backed out of the supplier's driveway, the contractor accidently hit the supplier's car, damaging it significantly. The supplier sued the contractor in federal court, asserting both a breach of contract claim and a negligence claim for the damage done to the supplier's car. The supplier's complaint seeks $71,000 in damages for breach of contract and $5,000 in damages to pay the cost of repairs to the car. Does the federal court have subject matter jurisdiction over the supplier's action? A. Yes, because the amounts of the supplier's two claims may be aggregated and thus meet the amount in controversy requirement. B. Yes, because the supplier's two claims are related and therefore may be aggregated to satisfy the amount in controversy requirement. C. No, because the supplier's two claims do not arise from the same transaction or occurrence and therefore may not be aggregated to satisfy the amount in controversy requirement. D. No as to the negligence claim but yes as to the breach of contract claim.

The court has subject matter jurisdiction because the two claims may be aggregated. Diversity of citizenship jurisdiction is available when (i) there is complete diversity of citizenship, meaning that each plaintiff is a citizen of a different state from every defendant; and (ii) the amount in controversy exceeds $75,000. Here, complete diversity exists because the case is one involving a supplier/plaintiff from State A against a contractor/defendant from State B. For the purposes of determining the jurisdictional amount, the plaintiff may aggregate all his claims that he has against a single defendant. As a result, the amount in controversy here is $76,000 ($71,000 for the breach of contract claim and $5,000 for the claim for damages to the automobile). (B) is incorrect both because the claims are not related factually and because it does not matter that they are not related. (C) is similarly incorrect because the claims need not be related. Aggregation is permitted regardless of whether the claims are legally or factually related to each other. (D) is incorrect because the amount in controversy is $75,000. The court would not have jurisdiction over the contract claim by itself. Furthermore, that answer choice ignores the possibility of aggregation.

A citizen of State A asserted a state law claim of $80,000 against a citizen of State B in the federal district court. The State B citizen has a state law claim against another citizen of State B for $90,000 that arose out of the same transaction or occurrence as the original complaint. As a result, the State B citizen brought a third-party action against that person. Does the court have subject matter jurisdiction over the State B citizen's claim in the third-party action? A. No, because the amount in controversy is insufficient. B. No, because there is no diversity of citizenship. C. Yes, because the requirements of diversity jurisdiction have been met. D. Yes, because the court has supplemental jurisdiction.

The court has subject matter jurisdiction over this claim under its supplemental jurisdiction powers. The prerequisites for diversity of citizenship do not exist, because both are residents of State B. However, the court would have supplemental jurisdiction over a third-party claim that arose from the same transaction or occurrence as the underlying claim. Although (B) is a true statement, it is incorrect because there is supplemental jurisdiction. (A) is factually incorrect. (C) is incorrect because, as stated, diversity of citizenship is lacking.

Four investors, all of whom are American citizens, own as a partnership a chain of 15 car dealerships in a number of states. Two of the investors live in State A, one lives in State B, and one lives in State C. The investors leave the day-to-day operation of each dealership to a manager that the partnership employs. The investors leave the management of the entire chain of dealerships and the day-to-day operation of the partnership to several key officers that it employs. The officers operate out of the partnership's largest dealership, which is in State D. A customer of the State D dealership sued the partnership in federal district court in State D, alleging fraud and breach of contract arising from her purchase of a car, and claiming, in good faith, damages exceeding $75,000. The customer is a citizen of State D. Does the federal district court in State D have subject matter jurisdiction over the customer's action against the partnership? A. Yes, because the federal district court is located in State D and not another state. B. Yes, because the plaintiff customer is a citizen of State D while the defendant partnership is a citizen of State A, State B, and State C. C. No, because the plaintiff customer is a citizen of State D and the defendant partnership is also a citizen of State D, the state where its principal place of business is located. D. No, because federal courts do not have subject matter jurisdiction over local transactions that take place entirely in one state.

The court has subject matter jurisdiction. Diversity of citizenship jurisdiction is available when (i) there is complete diversity of citizenship, meaning that each plaintiff is a citizen of a different state from every defendant; and (ii) the amount in controversy exceeds $75,000. A natural person's citizenship is the state that is the person's permanent home. A partnership is a citizen of each state of which one of its partners is a citizen, both limited and general. Here, the plaintiff is a citizen of State D, and the partners are citizens of State A, State B, and State C. Thus, complete diversity exists, and the amount in controversy is stated to exceed $75,000. As a result, diversity of citizenship jurisdiction exists. (A) is incorrect because diversity would be present in any federal court. Subject matter jurisdiction would thus exist in any federal court. (C) is also incorrect. A corporation's principal place of business is one of its possible citizenships-every state in which the corporation was incorporated being the other possible citizenships. The same rule does not apply to partnerships; a partnership takes on the citizenships of its partners. (D) is an incorrect statement of the law. The fact that the transaction was local does not preclude subject matter jurisdiction founded on diversity of citizenship jurisdiction.

A pedestrian was injured in a car accident involving two cars. The pedestrian filed a negligence action against both drivers in federal district court, seeking to hold them jointly and severally liable for $500,000. The pedestrian is a citizen of State A, and both drivers are citizens of State B. The first driver then asserted a cross-claim against the second driver, claiming that the second driver's negligence was the sole cause of the accident and seeking to recover from the second driver $15,000 for the damage done to the first driver's car. Does the federal court have subject matter jurisdiction over the cross-claim? A. No, because the cross-claim is between two citizens of the same state. B. No, because the amount of the cross-claim is too small. C. No, because there is no supplemental jurisdiction over the cross-claim unless it is related to a federal question claim asserted in the same action, and there is no federal question claim. D. Yes, the court has supplemental jurisdiction over the cross-claim.

The court has supplemental jurisdiction over the cross-claim. Generally, every claim in federal court must have a basis for federal subject matter jurisdiction. There are three main flavors of federal subject matter jurisdiction-diversity of citizenship jurisdiction, federal question jurisdiction, and supplemental jurisdiction. Diversity of citizenship jurisdiction is available when (i) there is complete diversity of citizenship, meaning that each plaintiff is a citizen of a different state from every defendant; and (ii) the amount in controversy exceeds $75,000. Here, the case is properly in federal court because diversity jurisdiction exists for the underlying claim (i.e., the claim by the pedestrian against both drivers), given that the pedestrian is from State A and the two drivers are from State B, and the amount in controversy is $500,000. The cross-claim by the first driver against the second driver for the damage to his car cannot invoke diversity jurisdiction because complete diversity does not exist (they are both from State B) and the amount claimed is only $15,000. However, the first driver might invoke supplemental jurisdiction to have the claim against the second driver heard in federal court. When the federal court has subject matter jurisdiction over one claim, it has discretion to exercise supplemental jurisdiction over related claims that derive from the same common nucleus of fact and are such that a plaintiff would ordinarily be expected to try them in a single judicial proceeding. In the instant case, the claim of the pedestrian vs. the first driver and the cross-claim of the first driver vs. second driver are derived from the same accident, and thus both are derived from the same common nucleus of fact. Thus, the first driver may use supplemental jurisdiction to have his cross-claim against the second driver heard in federal court. (A) is incorrect for the reasons described above. There are restrictions on invoking supplemental jurisdiction if its application by a plaintiff would be contrary to the complete diversity requirement in a diversity jurisdiction case. In terms of third-party practice, a plaintiff may not invoke supplemental jurisdiction to make claims against an impleaded party from the plaintiff's state. However, cross-claims by a defendant are not listed among the restrictions, and thus supplemental jurisdiction is available in a cross-claim by one defendant against another even though they are from the same state. (B) is incorrect because, although the claim does not satisfy the minimum amount in controversy for diversity actions, it can be heard in federal court under the court's supplemental jurisdiction, as explained above. (C) is incorrect because, subject to certain restrictions, supplemental jurisdiction is available when the underlying claim is in federal court via diversity of citizenship jurisdiction.

An author from State A filed a claim in federal court sitting in State B against a publisher headquartered in State B. The complaint alleged that the publisher plagiarized a portion of the author's book and asserted both a copyright infringement claim under federal law and an unfair business practices claim under an applicable State B statute. At trial, the publisher presented evidence that the author never filed the copyright infringement claim with the appropriate federal agency, as required by the federal statute, thereby invalidating the copyright infringement claim. The claim for unfair business practices, however, was still capable of obtaining a favorable verdict. The publisher moved for dismissal of the state law claim as well. How should the federal court rule? A. The court must remand the state law claim to state court because there is no federal subject matter jurisdiction without the copyright infringement claim. B. The court must dismiss the state law claim, because there is no federal subject matter jurisdiction without the copyright infringement claim. C. The court should, in its discretion, retain jurisdiction over the state law claim because the trial has begun. D. The court should, in its discretion, dismiss the state law claim because the jury has not yet begun to deliberate.

The court may dismiss or hear the state law claim in its discretion, but will likely retain jurisdiction over it. When a claim is in federal court under federal question jurisdiction, and the plaintiff has a state law claim against the defendant that cannot invoke diversity jurisdiction, the federal court has discretion to exercise supplemental (pendent) jurisdiction over the state law claim if the federal and state claims derive from a common nucleus of operative fact and are such that a plaintiff would ordinarily be expected to try them all in one judicial proceeding. The court may continue to exercise supplemental (pendent) jurisdiction over the state claim even though the federal claim is dismissed on the merits. However, the state claim should probably also be dismissed (without prejudice) if the federal claim is dismissed before trial. Here, although the federal copyright claim was invalid, it was deemed to be so during trial, before a verdict was rendered. Since the case is currently being tried, the court will likely hear the state claim for the sake of judicial economy. (B) is wrong, because, as stated above, the federal court has the discretion under supplemental jurisdiction to hear the state claim. (A) is wrong for the same reason, and also because remand can only occur when the action was commenced in state court and then removed to federal court. (D) is wrong because, although the decision is within the court's discretion, the start of the trial is the point at which a federal court will usually retain jurisdiction over supplemental state law claims for the sake of judicial economy.

A manufacturer sued a department store for breach of contract, alleging that the department store failed to pay for a shipment of watches delivered by the manufacturer to the department store in July of that year. At trial, the manufacturer introduced evidence regarding the department store's failure to pay for the watches. In addition, it introduced evidence that the department store failed to pay for a shipment of necklaces, also delivered that July pursuant to a separate contract. At the close of the manufacturer's case, the department store introduced evidence showing that both the watches and the jewelry were defective. The jury returned a verdict for the manufacturer, awarding the manufacturer damages for breach of both contracts. The manufacturer then immediately moved to amend its complaint to conform to the evidence introduced at trial. Should the court allow the amendment? A. Yes, because the department store impliedly consented to the introduction of the evidence. B. Yes, because the parties entered into the contracts at approximately the same time. C. No, because the motion should have been made before the verdict was rendered. D. No, because the defendant must have had knowledge of the claim when the complaint was served.

The court should allow the amendment. An issue not raised by the pleadings is treated as if raised in the pleadings if it is tried by the parties' express or implied consent. In such a case, a party may move-at any time, even after judgment-to amend the pleadings to conform them to the evidence and to raise the unpleaded issue. Here, the department store impliedly consented to the introduction of evidence relating to the jewelry contract. Not only did it fail to object to the evidence, it introduced evidence to show that the jewelry was defective. Thus, the manufacturer was entitled to move to amend its complaint to conform to the evidence introduced at trial. (B) is incorrect because the relevant determination is whether the party impliedly consented to the trial of the issue, not whether the issue was related to an issue raised by the pleadings. (C) is incorrect because a motion to amend a pleading to conform to the evidence introduced at trial may be made at any time, even after judgment. (D) is incorrect because the defendant need not know about the claim when the complaint was served.

A resident of State A sued a resident of State B in federal district court in State B for breach of contract. Jurisdiction was based on diversity of citizenship. The plaintiff alleged that the contract was entered into in State C and was to be performed in State D. The plaintiff further alleged that the defendant failed to perform. While hearing this case, what substantive law should the federal district court apply? A. The law that the State D state court would apply. B. The law that the State C state court would apply. C. The law that the State B state court would apply. D. The law that the federal district court believes most logically applies.

The court should apply the law that the State B state court would apply. In a diversity case, the federal court applies the law that would be applied by the courts of the state in which the federal court is located. This includes the state's choice of law rules. In the instant case, the federal court sitting in State B may well, as an end result, apply the law of State D or State C, but if it does so, it will be because State B's choice of law rules require such a result. (A) and (B) are incorrect because the answers imply that the federal court will apply State D's or State C's choice of law rules, which is incorrect. (D) is incorrect because the federal court cannot ignore the law the state would apply even though it finds that law to be unsatisfactory.

A plaintiff from State A filed a breach of contract action in a federal court in State B. The defendant in the action is a State B corporation. The corporation's president and registered agent, the only officer having express authority to receive service of process on behalf of the corporation, was out of the country for several months. State B's rules of civil procedure authorize service on a corporation by serving any "officer, managing or general agent, or any agent authorized to receive service." State B's rules also authorize service of process by certified mail. The plaintiff thus sent the summons and complaint to the defendant corporation's only vice president, who resides in State B. The vice president signed the certified mail return receipt. If the corporation files a motion to dismiss the action for improper service of process, how should the court rule? A. Deny the motion, because service of process was proper. B. Deny the motion, because the defendant waived service of process by allowing its only agent authorized to receive service to leave the country. C. Grant the motion, because, despite what state rules allow, service on a corporation sued in federal court must be on the officer or agent of the corporation who is authorized by the corporation to receive service on behalf of the corporation. D. Grant the motion, because, despite what state rules allow, service on a corporation sued in federal court may not be made by certified mail.

The court should deny the motion to dismiss because service of process was proper. The federal rules do not expressly authorize service on a corporation by service on individuals by certified mail. However, the rules do authorize service pursuant to the rules used by the state in which the federal court sits and in which service takes place. Thus, the plaintiff here may use certified mail as a means of serving process. (B) is incorrect because the fact that the expressly authorized agent left the country is not really relevant to service of process. Other methods are authorized. Furthermore, the fact that the authorized agent was out of the country might be good cause to have the period for service of process extended. (C) is incorrect because, as explained above, a litigant in federal court may use state rules in which the federal court is sitting for serving process. Furthermore, Federal Rule 4(h)(1) authorizes service "by delivering a copy of the summons and the complaint to an officer, a managing or general agent, or any other agent authorized by appointment or by law to receive service of process and-if the agent is one authorized by statute and the statute so requires-by also mailing a copy of each to the defendant." (D) is incorrect for much of the same reason as (C). A litigant in federal court may use state rules in which the federal court is sitting for serving process.

A plaintiff filed a single cause of action against a defendant on a claim based on products liability in state court. The plaintiff contends that a space heater that he was using during the winter short-circuited, causing a fire that burned down his house. In July, the plaintiff's case against the defendant went to trial, and the jury determined that the fire was caused by faulty wiring in the plaintiff's home. The following September, the plaintiff filed a breach of warranty action against the defendant in federal court, properly invoking the court's diversity jurisdiction. The defendant, after properly being served with process, files an answer with an affirmative defense of claim preclusion as his first pleading or motion. The defendant, at an appropriate time, then files a motion for summary judgment based on the assertion of claim preclusion. How should the court rule on the defendant's motion? A. Deny the motion, because the application of claim preclusion is a factual issue for the jury to determine. B. Deny the motion, because the plaintiff's breach of warranty theory is not the same cause of action as the plaintiff's products liability claim. C. Grant the motion, because claim preclusion applies to bar the claim. D. Grant the motion, because the defendant, by raising the issue in his first pleading or motion, did not waive the defense of claim preclusion.

The court should grant the motion on claim preclusion grounds. Before claim preclusion (also called res judicata) applies, it must be shown that (i) the earlier judgment is a valid, final judgment "on the merits"; (ii) the cases are brought by the same claimant against the same defendant; and (iii) the same "cause of action" (or "claim") is involved in the later lawsuit. While various tests have been used to define "cause of action," the modern approach is to require assertion of all claims arising out of the same transaction or occurrence that is the subject matter of a claim asserted by the claimant. Here, the products liability claim was determined in the defendant's favor by a jury. Thus, there is a valid, final judgment. The same plaintiff is suing the same defendant, and the same claim is involved, as both the products liability and breach of warranty claims arise out of the same transaction or occurrence-the alleged malfunction of the space heater that burned down the plaintiff's home. Thus, claim preclusion applies to bar the claim. (Note that when the plaintiff loses the earlier case, claim preclusion "bars" further litigation. When the plaintiff wins the earlier case, the different theories "merge" to prevent further litigation.) (A) is incorrect because claim preclusion is not a factual issue for the jury to determine. It is a legal issue for the judge to determine. (B) is incorrect because, as explained above, a later claim is deemed to be the same claim if it arises from the same transaction or occurrence as the earlier claim. (D) is incorrect because a defense of claim preclusion is not necessarily waived if it is not included in the defendant's first pleading or motion. A judge would have discretion to allow a defendant to amend his answer to include an omitted defense, and she will likely do so when no prejudice to the defendant results. (Leave to amend is freely granted when justice so requires.)

A State A citizen and a State B citizen were in an automobile accident in State B. The State B citizen filed a negligence action for $500,000 against the State A citizen in a federal district court located in State B. The State A defendant would prefer to litigate the case in a State B state court. The State A defendant thus filed a notice of removal, seeking to transfer the case to a State B state court. Should the federal court grant the motion? A. Yes, because federal diversity jurisdiction is not needed to protect the State A citizen from the potential bias of State B courts if the State A citizen requests that the matter be litigated in the State B state courts. B. Yes, because tort actions arising from accidents in State B should be litigated in State B state courts. C. No, because removal to state court is not available for cases that are properly filed in federal court and that are within the federal court's subject matter jurisdiction. D. No, because removal to state court is not available when one of the parties is a citizen of the state in which the action is pending.

The court should not grant the motion. Under 28 U.S.C. section 1441, a defendant may remove an action that could originally have been brought in the federal courts, based on either a federal question being presented or on diversity of citizenship. However, there is no similar provision that allows a case initially filed in federal court to be moved to state court. (An action that was wrongfully removed from state court to federal court may be remanded back to state court, but that procedure is not applicable here because the case was initially filed in federal court.) (A) is incorrect. As stated, although one goal behind diversity jurisdiction is to lessen an in-state prejudice against out-of-state defendants, the lack of that potential prejudice does not provide a basis for moving a case from federal court to state court. (B) is an incorrect statement of the law. A case based on diversity jurisdiction may include a tort action that arose in the jurisdiction. (D) is an incorrect statement of the law. An in-state defendant is prevented from removing a case to federal court based on diversity. The fact that the plaintiff may be a citizen of the state in which the state case was filed does not prevent removal.

A bar prep company discovered that its copyrighted content was being used in an online simulated exam that its competitor was administering in a few days. The company filed a petition for an ex parte order to direct the competitor to immediately remove that content from its website. The company submitted an affidavit specifying why immediate and irreparable injury will result if the exam is administered, and offered to provide security for any costs or damages incurred by the competitor if it was determined that the order was wrongfully issued. Should the court issue the order? A. Yes, because the company submitted an affidavit with specific facts showing immediate and irreparable injury. B. Yes, because the competitor will receive notice of the order once it is issued by the court. C. No, because a court cannot issue an injunction unless the adverse party has notice of the hearing. D. No, because the company has not provided sufficient certification for obtaining an ex parte order.

The court should not issue the ex parte order. A temporary restraining order may be granted by a court when it is necessary to prevent irreparable injury to a party, and the injury will result before a preliminary injunction hearing can be held. As a general rule, notice of the hearing for the issuance of the order must be given before it is issued. However, a court may grant an ex parte temporary restraining order without notice of the hearing to the adverse party if the moving party does the following: (i) gives specific facts in an affidavit or a verified complaint to establish that immediate and irreparable injury will result to the moving party before the adverse party can be heard in opposition; (ii) certifies in writing all efforts it made to give notice to the adverse party and why notice should not be required; and (iii) provides some security to pay for any costs and damages incurred by the adverse party if it is wrongfully enjoined or restrained. Here, the company alleged irreparable injury and offered to provide security, but nothing in the facts indicates that it certified efforts to give notice to the competitor or why notice should not be required. The facts do not establish that it is impossible to provide notice to the competitor before a restraining order is issued. (A) is incorrect because the company's affidavit is not enough to justify the issuance of an ex parte order, as discussed above. (B) is incorrect because, while actual notice of the order also is required, notice of the hearing is required as well unless the three requirements stated above are met. (C) is incorrect because it is too broad. A court may issue an ex parte order without prior notice to the adverse party if the three requirements stated above are met.

A pedestrian sued a driver for personal injuries in federal court, properly invoking diversity of citizenship jurisdiction. In the complaint, the pedestrian alleged that the driver ran a red light and struck the pedestrian while the pedestrian was in the crosswalk. Concurrent with the accident, a police report was prepared on which the name and address of a witness to the accident was listed, but neither party requested a copy of the report from the police department. Thus, when the pedestrian submitted an interrogatory to the driver for the names and addresses of persons with knowledge of the accident known to the driver, the driver truthfully omitted the name of the witness. When asked during his deposition whether he knew of any witnesses, the driver again truthfully answered "I don't know of any." At trial, the jury found for the driver. In one of the special interrogatories answered by the jury, the jury found that the driver had the green light and that the pedestrian was crossing against the light. Six months and a day after a final judgment was rendered in favor of the driver, the pedestrian's attorney was contacted by the witness, who stated that the driver ran a red light and that the pedestrian had the "walk" sign when he attempted to cross the road. The pedestrian's attorney immediately moved for relief from judgment based on newly discovered evidence, and the trial judge granted the motion. On appeal, what should the court do? A. Affirm the trial judge's decision, because whether to deny or grant a motion for relief from judgment is strictly a matter within the trial judge's discretion. B. Affirm the trial judge's decision, because the driver should have discovered the identity of the witness. C. Reverse the trial judge's decision, because more than six months had passed since the judgment was rendered when the trial judge granted the motion. D. Reverse the trial judge's decision, because the pedestrian could have discovered the witness's identity with reasonable diligence.

The court should reverse the trial judge's decision. On motion and just terms, the court may relieve a party from a final judgment or order on the following grounds: (i) mistake, inadvertence, surprise, or excusable neglect; (ii) newly discovered evidence that by due diligence could not have been discovered in time to move for a new trial; (iii) fraud, misrepresentation, or other misconduct of an adverse party; (iv) the judgment is void; (v) the judgment has been satisfied, released, or discharged; a prior judgment on which it is based has been reversed or otherwise vacated; or it is no longer equitable that the judgment should have prospective application; or (vi) any other reason justifying relief from the operation of the judgment. For grounds (i), (ii), and (iii), the motion must be made within a reasonable time not to exceed one year from the judgment; for the other grounds, the motion must be made within a reasonable time. (But remember that a lack of subject matter jurisdiction may be raised at any time.) Such a motion is left to the trial judge's discretion, and, on appeal, her decision will be reviewed on an "abuse of discretion" standard. Here, the motion would be based on newly discovered evidence. When deciding whether to grant or deny such a motion, the trial judge must determine whether the evidence could have been discovered with any amount of due diligence. Here, the name and address of the witness could have been easily discovered had the pedestrian's attorney simply requested a copy of the police report from the police department. This is such an obvious case of lack of due diligence on the part of a represented party that it would be an abuse of discretion to grant relief based on newly discovered evidence. (C) is incorrect because the time period for bringing a motion for relief from judgment based on newly discovered evidence is within a reasonable time, not to exceed one year from the judgment, not six months. (A) is incorrect because, although the decision is left to the trial judge's discretion, the exercise of discretion would be reviewed on appeal on an abuse of discretion standard, and here, the newly discovered evidence could have easily been discovered in time for trial with minimal preparation. Thus, the judge very likely abused discretion by granting the motion. (B) is incorrect. Although a party is required to disclose information within his possession upon a proper request (and after a proper search of his records), and to truthfully answer questions at a deposition, those requirements do not relieve the opposing party from conducting an investigation of her own case. In other words, the fact that the driver's attorney may have been grossly negligent in preparing his case does not relieve the failure of the pedestrian's attorney to discover the identity of the witness.

On August 1, the plaintiff, a resident of State A, sued two defendants in State A for personal injuries arising out of an automobile accident. One defendant is a citizen of State A while the other is a citizen of State B. The lawsuit claimed damages of $500,000. The plaintiff quickly reached a settlement agreement with the defendant from State A, and the court dismissed that defendant by order on August 16. The order is served on the remaining defendant on August 20. On September 18, the remaining defendant files a notice of removal with the court, which the plaintiff opposes. How should the court rule on the defendant's notice of removal? A. For the remaining defendant, because she filed her notice of removal within 30 days after she discovered the case had become removable. B. For the remaining defendant, because there are no time restrictions on removing a case to federal court. C. For the plaintiff, because a plaintiff has the right to choose his own forum. D. For the plaintiff, because more than 30 days have passed since the case became removable.

The court should rule for the remaining defendant. There are essentially two time restrictions on removal of a diversity case to federal court: (1) a case based on diversity must be removed within 30 days of the defendant's receipt of a copy of the paper (order, motion, etc.) that makes the case removable; but (2) in no event may the case be removed more than one year after it was commenced in state court. [28 U.S.C. §1446] Here, it is the 30-day time limit that is in question; i.e., specifically, whether the 30-day clock starts to tick on August 16 (when the case became removable) or on August 20 (when the defendant learned that the case became removable). As stated above, it is the latter. The clock starts to tick when the defendant learns by service of any paper that the case has become removable. Because she requested removal within this 30-day period, the court should rule in favor of the remaining defendant.

A consumer purchased a luxury automobile from a dealer on credit. After the consumer failed to make a number of the required payments, the dealer filed a civil action against the consumer in federal district court to recover the balance due on the account. The dealer properly served process on the consumer. Several months passed, and the consumer did not file any response to the complaint. The dealer then filed a motion asking the clerk of court to make an entry of default, and the clerk did so. What procedure should the dealer follow to obtain a default judgment against the consumer? A. File a motion to have the clerk of court enter the default judgment, and the clerk may do so without the consumer receiving any further notice of the motion. B. File a motion to have the clerk of court enter the default judgment, and the clerk may do so, provided the consumer receives additional notice of the motion for default judgment. C. File a motion to have the judge enter the default judgment, and the judge may do so without the consumer receiving any further notice of the motion for default judgment. D. File a motion to have the judge enter the default judgment, and the judge may do so, provided the consumer receives additional notice of the motion for default judgment.

The dealer should file a motion with the clerk of court. On request of the plaintiff, supported by an affidavit as to the amount due, the clerk may sign and enter judgment for that amount and costs against the defendant if: (i) the plaintiff's claim against the defaulted defendant is for a sum certain; (ii) the default was entered because the defendant failed to appear; (iii) the defaulted defendant is not an infant or incompetent person; and (iv) the damages amount requested is not greater than the amount requested in the complaint. The dealer's claim here meets these requirements, so the clerk may enter the default judgment, and no notice is required. (B) is incorrect because no further notice is required under these facts. (C) is incorrect because the clerk may enter the default judgment when the amount is for a sum certain. (D) is incorrect because no further notice is required under these facts and because the clerk may enter the default judgment.

A corporation filed a breach of contract action against another corporation in the United States District Court for the District of State A on the basis of diversity of citizenship jurisdiction. The action seeks $80,000 in damages. The plaintiff corporation was incorporated in State B and has its principal place of business in State C. The defendant corporation was incorporated in State C and has its principal place of business in State A. The defendant corporation filed a motion to dismiss the action for lack of subject matter jurisdiction. Will the defendant corporation's motion be granted? A. Yes, because both corporations are citizens of State C. B. Yes, because the defendant corporation is a citizen of the state in which the action was filed. C. No, because the plaintiff corporation is a citizen of State C and the defendant corporation is a citizen of State A. D. No, because, while both corporations are citizens of State C, the plaintiff corporation is also a citizen of State B and the defendant corporation is also a citizen of State A.

The defendant corporation's motion will be granted. For there to be diversity of citizenship jurisdiction, each plaintiff must be a citizen of a different state from every defendant, and the amount in controversy must exceed $75,000. A corporation is deemed to be a citizen of every state in which it was incorporated and the one state in which it has its principal place of business. Here, the plaintiff corporation is a citizen of State B (in which it was incorporated) and State C (its principal place of business), while the defendant corporation is a citizen of State C (in which it was incorporated) and State A (its principal place of business). Thus, the two corporations share citizenship in State C, and complete diversity does not exist between the two corporations. (B) is incorrect. Whether a defendant is a citizen of a home state is relevant for the purposes of removal. (An in-state defendant may not remove a case from state court to federal court if the defendant is a citizen of the state in which the case was filed.) That fact, however, does not prohibit the filing of a case in federal court based on diversity. (C) is incorrect. The answer implies that only the principal place of business is considered when determining whether diversity exists, which is not the case. (D) is incorrect because it incorrectly states the rule for determining the citizenship of a corporation. Corporations may have multiple citizenships, each of which is considered when determining whether complete diversity exists.

A citizen of State A filed a breach of contract action against a citizen of State B in a State A state trial court. The State B defendant timely and properly removed the action to the United States District Court for the District of State A. The defendant then filed a motion to dismiss the action based on insufficient service of process. Following a hearing, the court found that service was proper and denied the motion. The defendant then filed her answer, responding to the merits of the complaint and asserting that the case should be dismissed on the grounds that another action was pending between the same parties for the same cause in a State B state court. The State A Rules of Civil Procedure provide that a party waives the right to seek dismissal on that ground if the party files a pre-answer motion to dismiss and does not assert that ground in the motion. Should the federal court hold that the defendant has waived the right to seek dismissal based on the pendency of the same cause in another court? A. Yes, because this action was commenced in a State A state court, so the federal court must apply State A law. B. Yes, because this action is in a federal court in State A on the basis of the court's diversity of citizenship jurisdiction, so the federal court must apply State A law. C. No, because this issue is unlikely to affect the outcome of the action or to create forum-shopping, so the federal court should apply federal law, under which the defendant has not waived this defense. D. No, because this issue is governed by the Federal Rules of Civil Procedure, under which the defendant has not waived this defense.

The defendant has not waived this defense. Unlike the state rule here, Federal Rule of Civil Procedure 12 does not require the defendant to raise the defense that another action is pending between the parties in the first responsive pleading. While the federal court exercising diversity of citizenship subject matter jurisdiction must apply state substantive law (here, State A law), applicable Federal Rules of Civil Procedure supersede state law in federal court as long as the rule comports with the requirements of the Rules Enabling Act (i.e., the rule governs practice and procedure and does not modify or abridge substantive rights). (A) and (B) are incorrect because they are too broad. The federal court need not apply State A procedural law. (C) is incorrect because, if the rule in question is on point and clearly procedural, it is simply applied. There is no need to determine if its application would affect the outcome of the case.

After filing her complaint in federal district court, a plaintiff mailed to a defendant by ordinary first-class mail the following: duly executed copies of the complaint and summons, a request that the defendant waive service of process, two copies of a waiver-of-service form, and an addressed, prepaid return envelope. The defendant signed the waiver-of-service form and returned a copy to the plaintiff. Fifty days after the plaintiff mailed the forms to the defendant and 25 days after the defendant returned the signed form to the plaintiff, the plaintiff filed a motion for entry of default and a default judgment. The following day, the defendant filed and served his answer. Was the defendant's answer timely? A. No, because the defendant may no longer file an answer once the time for filing a response to a complaint has expired and the plaintiff has filed a motion for default judgment. B. No, because the Federal Rules of Civil Procedure require a defendant to file a response to a complaint within 21 days after being served with process. C. Yes, because the defendant filed his answer less than 28 days after he returned the waiver of service form. D. Yes, because a defendant who waives service of process is allowed additional time in which to respond to the complaint, and the time had not yet expired.

The defendant's answer was timely. Although a defendant typically must serve an answer or other appropriate response within 21 days after being formally served with process, a defendant who waives service of process is allowed 60 days from the date on which the plaintiff mailed the summons and complaint in which to serve a response. (A) is incorrect for a few reasons. First, as explained above, the time for filing had not expired. But even if it had, if the defendant files an answer after a motion for default judgment is filed but before the default is entered, the court must determine whether the defendant has good cause for filing a late answer as the court has discretion to extend the time for answering. (B) is incorrect. As explained above, when the defendant waives service of process, the period for answering the complaint is extended to 60 days. (C) is not as good an answer as (D) because the period for answering is not measured from the date the waiver of service is returned but rather from the date it is sent, as explained above.

A plaintiff brought an action in a state court against a defendant, a city police chief in the state, alleging civil rights violations under 42 U.S.C. section 1983. The defendant moves to dismiss the state suit on the ground that the action must be brought in federal court because a federal question is involved. Should the court grant the defendant's motion to dismiss? A. Yes, because the federal court has exclusive jurisdiction over the action. B. Yes, but the plaintiff may defeat dismissal by adding a state law claim. C. No, because the federal courts and state courts have concurrent jurisdiction over the action. D. No, but the defendant may remove to federal court if he has a defense based on federal law.

The defendant's motion will be denied. The federal courts and the state courts have concurrent jurisdiction over most types of actions. The few areas over which the federal courts have exclusive jurisdiction include bankruptcy cases, patent and copyright cases, antitrust cases, and a few other less common types of actions. (A) is incorrect for the reasons discussed above. (B) is incorrect because the appropriate remedy for a defendant would be to have the case removed to federal court. Furthermore, a defendant would be able to remove notwithstanding the state law claim; a court could invoke its supplemental jurisdiction to have a pendent state claim heard with claims based on federal law. (D) is incorrect. A defense based on federal law would not enable a defendant to remove the case to federal court.

The plaintiff in a negligence case was injured in a car accident with the defendant. The plaintiff's attorney retained a physician to examine the plaintiff and to testify regarding the plaintiff's injuries. After the physician examined the plaintiff, the plaintiff's attorney discussed the physician's findings with the physician. The plaintiff's attorney and the physician also discussed what the physician should include in the expert witness report that will be provided to the defendant. The defendant then conducted a deposition of the physician. During the deposition, the defendant's attorney asked the physician to describe what was said in his conversations with the plaintiff's attorney. The plaintiff's attorney objected to those questions. Are the defendant's questions regarding the plaintiff's attorney's discussions with the physician properly subject to discovery? A. No, because the communications between the plaintiff's attorney and the physician are protected from discovery under the work product doctrine. B. No, because the communications between the plaintiff's attorney and the physician are protected by the attorney-client privilege. C. Yes, because the attorney does not represent the physician and therefore the communications between the plaintiff's attorney and the physician are not privileged. D. Yes, because the communications between the plaintiff's attorney and the physician are relevant to determine the basis for opinions and statements in the physician's expert report.

The discussions are not subject to discovery. Draft reports and draft disclosures of "trial" experts are work product. Confidential communications between such experts and counsel for the party are also generally protected under the work product doctrine, except for communications relating to the expert's compensation or to facts or data the attorney provided to the expert. (B) is incorrect because, although communication between the plaintiff, attorney, and expert could conceivably fall under the attorney-client privilege, it is not as certain that communications between only the attorney and the expert would fall under the attorney-client privilege. However, the Federal Rules specifically exempt such communication from discovery as work product (and it is discoverable only in extraordinary circumstances, as explained below). As a result, (A) is a better choice than (B). (C) is incorrect because, although the discussions might not be protected under the attorney-client privilege, they will be protected as work product, as described above. (D) is incorrect because work product is specifically exempted from discovery unless the discovering party can show substantial need and undue hardship in securing similar material from other sources, despite the fact that the material may fall within the technical definition of the scope of discovery (any nonprivileged matter that is relevant to any claim or defense of a party and proportional to the needs of the case).

On January 15, a patient sued his doctor and his surgeon in State A state court for medical malpractice. All acts of malpractice took place in State A. The patient and the surgeon are citizens of State A; the doctor is a citizen of State B. Fifteen months later, it was learned after extensive discovery that the surgeon was only peripherally involved in the patient's treatment and was in no way negligent, so the patient dismissed the cause of action against the surgeon. Two weeks later, the doctor seeks to remove the case against him to federal court in State A, alleging diversity jurisdiction. May the doctor successfully remove the case to a federal district court? A. Yes, because removal was sought within 30 days of the date that the doctor first learned that the case had become removable. B. Yes, because removal was sought within one year of the case becoming removable. C. No, because a case may not be removed to federal court more than one year after the action was commenced. D. No, because a defendant may not seek removal if the cause of action accrued in the forum state.

The doctor may not remove the case. If a diversity action is not initially removable but later becomes removable (as by dismissal of a nondiverse defendant), it may not be removed more than one year after it was commenced in state court. (A) is incorrect. In all cases, the defendant has 30 days after a case becomes removable to file a notice of removal; however, for diversity cases only, removal must also occur within one year from the date the case is commenced. (B) is incorrect. The one-year limit begins when the action is commenced, not when the case first became removable. (D) is incorrect. There is no such rule.

A citizen of State A ran a red light at a traffic intersection, striking a pedestrian, a citizen of State B, who was lawfully in the middle of the crosswalk at the time. The accident occurred in State B. Despite the pedestrian's extensive injuries, the driver was more concerned with the damage to his new automobile. The driver sued the pedestrian in a state court of State B for $90,000 for damage to his car. The driver promptly lost, and then decided to bring suit in federal court, claiming diversity jurisdiction. May the driver successfully bring his claim in federal court? A. Yes, because the requirements for diversity jurisdiction are met. B. Yes, because a state court decision does not bind a federal court. C. No, because all claims merged with the state court case. D. No, because the claim is barred.

The driver will not be successful because the claim is barred. Bar occurs when a plaintiff loses. The adverse judgment "bars" the plaintiff from relitigating the cause of action. (A) is incorrect because, although the requirements of diversity jurisdiction are met, the driver will not be successful because the claim is barred, as discussed above. (B) is incorrect. Federal courts will give the same preclusive effect of a state court decision that the state that issued it would; e.g., here, a federal court would give the State B state court decision the same preclusive effect that another State B state court would. (C) is incorrect. Merger occurs when a plaintiff wins; his cause of action is said to "merge" into the judgment such that he cannot relitigate the cause of action later.

An employee filed an action against his employer in federal district court, claiming unlawful age discrimination in employment. The employee did not like the judge who was assigned to the case because the employee perceived the judge to be antagonistic to employment discrimination claims. Two weeks after filing the action and before the employer served its answer to the complaint, the employee voluntarily dismissed the action. Several months later, the employee filed an identical action in the same court. Is the employee's claim barred because of the prior dismissal? A. Yes, because a voluntary dismissal operates as adjudication on the merits and bars subsequent assertions of the same claim. B. Yes, because, while voluntary dismissals for valid reasons will not preclude subsequent assertions of a claim, the employee's reason for the voluntary dismissal in this instance is invalid and thus bars the subsequent assertion of the claim. C. No, unless the employer properly asserts the affirmative defense of claim preclusion. D. No, because the prior dismissal was without prejudice since the employee had not previously dismissed the same claim.

The employee's claim is not barred because of the prior dismissal. A plaintiff may of right dismiss an action one time without prejudice before the defendant serves its answer. (A) is incorrect because a voluntary dismissal is not an adjudication on the merits. (B) is incorrect because a plaintiff may voluntarily dismiss a case for any reason. (C) is incorrect because claim preclusion applies when there has been a valid, final judgment on the merits. A voluntary dismissal is not a final judgment on the merits; therefore the employer cannot properly assert the defense of claim preclusion.

A homeowner hired an exterminator company to treat an existing house to protect it from termite infestation. The exterminator determined that it had to remove dirt around part of the foundation in order to treat the house properly. The exterminator thus hired a subcontractor to remove the dirt with a backhoe. While removing the dirt, the subcontractor damaged the foundation of the house. The homeowner filed an action against the exterminator in federal district court, seeking $90,000 in damages. May the exterminator properly maintain a third-party claim against the subcontractor for any liability the exterminator may have to the homeowner? A. Yes, because third-party claims are permitted whenever they arise from the same transaction or occurrence as the original claims in an action. B. Yes, because the exterminator is claiming that the subcontractor is liable for the exterminator's liability to the plaintiff. C. No, unless the exterminator seeks and obtains the court's permission. D. No, because this claim does not meet the requirements for third-party practice.

The exterminator may properly maintain a third-party claim against the subcontractor. A defending party may implead a nonparty if the nonparty is or may be liable to the party for any part of a judgment that the plaintiff may recover against it. Because the exterminator is claiming that the subcontractor is liable for the exterminator's liability to the plaintiff, it may implead the subcontractor. (D) is therefore incorrect. (A) is too broad. It is not sufficient that the third-party claim merely arise from the same transaction or occurrence. (C) is incorrect because a defendant does not need the court's permission to bring a third-party claim.

A seller entered a contract with a buyer under which the buyer agreed to purchase certain corporate securities for $500,000. The buyer refused to go through with the purchase, contending that the entire transaction was void because of federal securities statutes. The seller thus sold the securities to another party but was able to sell them for only $420,000. The seller filed an action against the buyer in federal district court, alleging that the transaction did not violate federal securities statutes and seeking $80,000 for the buyer's breach of contract. The seller and the buyer are both citizens of the same state. Does the federal court have subject matter jurisdiction over the action? A. Yes, because the action raises the federal issue of whether the transaction violated federal securities statutes. B. Yes, because the amount in controversy exceeds the required minimum for purposes of federal question jurisdiction. C. No, because the buyer and seller are citizens of the same state and the action does not arise under federal law. D. No, because the amount in controversy for purposes of federal question jurisdiction is not sufficient.

The federal court does not have subject matter jurisdiction because neither federal question jurisdiction nor diversity of citizenship jurisdiction exists under these facts. Diversity of citizenship jurisdiction is available when (i) there is complete diversity of citizenship, meaning that each plaintiff is a citizen of a different state from every defendant; and (ii) the amount in controversy exceeds $75,000. Here, since the seller and buyer are both from the same state, complete diversity does not exist. Federal question jurisdiction is available when the plaintiff, in his well-pleaded complaint, alleges a claim that arises under federal law. Anticipation of a federal defense or the fact that federal law is implicated by the plaintiff's claim do not give rise to federal question jurisdiction; the plaintiff's claim must arise under federal law. Here, the plaintiff is anticipating that the defendant might raise federal securities laws as a defense, which is not enough to give rise to federal question jurisdiction. (A) is incorrect because the federal question is raised by the anticipation of a federal defense, which is not sufficient to give rise to federal question jurisdiction, as described above. (B) and (D) are incorrect. Both imply that there is a required jurisdictional minimum for federal question jurisdiction. Federal question jurisdiction does not require that any particular amount be in controversy.

Under the doctrine of issue preclusion (collateral estoppel), a judgment in a prior case will be sufficient to bind the plaintiff or defendant in subsequent actions on different causes of action as to issues that were: A. Decided by the judge B. Actually litigated and essential to the judgment C. Decided by a jury

Under the doctrine of issue preclusion (also called collateral estoppel), a judgment binds the plaintiff or defendant in subsequent actions on different causes of action as to issues that were actually litigated and essential to the judgment. It is not sufficient merely that the issue was decided by a judge; the issue still must be one that was actually litigated and essential to the judgment. Neither is it sufficient that the issue was decided by the jury; it still must be one that was actually litigated and essential to the judgment.

A driver was killed in a two-car accident in State A. The deceased driver was a citizen of State B, and the driver of the other car is a citizen of State A. The executor of the deceased driver's estate filed a negligence action in federal district court on behalf of the deceased driver against the State A driver, seeking $250,000. The executor is a citizen of State A. Does the federal court have diversity of citizenship jurisdiction over this action? A. No, because the executor and the defendant are both citizens of State A. B. No, because federal courts will not exercise diversity of citizenship jurisdiction over probate matters. C. Yes, because the executor of the plaintiff's estate is a citizen of State A and the deceased plaintiff is a citizen of State B. D. Yes, because the deceased plaintiff was a citizen of State B and the defendant is a citizen of State A.

The federal court has diversity of citizenship jurisdiction. Diversity of citizenship jurisdiction is available when (i) there is complete diversity of citizenship, meaning that each plaintiff is a citizen of a different state from every defendant; and (ii) the amount in controversy exceeds $75,000. A natural person's citizenship is the state that is the person's permanent home. For diversity purposes, a legal representative of a decedent assumes the state citizenship of the decedent. Here, one driver (the decedent) was a citizen of State B, whereas the other driver (the defendant) is a citizen of State A. Thus, complete diversity exists, and the $250,000 alleged (presumably in good faith) in the complaint satisfies the amount in controversy requirement. (A) is incorrect because, as explained, the executor assumes the state citizenship of the decedent, and the executor's state citizenship is disregarded. The fact that the executor may be from the same state as a defendant does not destroy complete diversity. (B) is incorrect. Although it is true that federal courts will not probate a decedent's estate, the exception is narrow. To fall within this exception for diversity of citizenship jurisdiction, the claim asserted must involve the actual probate or annulment of a will or seek to reach property in the custody of a state probate court. (C) is incorrect. There is no requirement for complete diversity between a decedent and his executor when the executor is representing the decedent in a diversity action.

The plaintiff, a citizen of State A, filed suit against the defendant, also a citizen of State A, in federal district court, alleging that the defendant had failed to perform a contract to provide 1,000 fully automatic machine guns. The defendant claimed that a recently enacted federal statute made the manufacture of fully automatic machine guns illegal. Does the federal district court have subject matter jurisdiction? A. No, because no federal question has been presented and the requirements of diversity of citizenship jurisdiction have not been met. B. No, unless the plaintiff moves to another state after filing suit. C. Yes, because a federal statute has been enacted, thereby presenting a federal question. D. Yes, if the value of 1,000 fully automatic machine guns exceeds $75,000.

The federal district court does not have diversity jurisdiction because both parties are citizens of State A. The court does not have federal question jurisdiction because the recently enacted federal statute arises only in anticipation of the defendant's defense. This is insufficient to confer federal question jurisdiction. (Rationale: If, for example, the defendant relied on some other defense or defaulted instead of defending on the basis of the new federal statute, no federal question would ever be involved in the case. [See Louisville & Nashville Railroad v. Mottley, 211 U.S. 149 (1908)]) (B) is incorrect because citizenship of the parties is determined at the time suit is filed. Assuming the $75,000 jurisdictional requirement could be met, the plaintiff would have had to move prior to filing suit to be able to invoke diversity of citizenship jurisdiction. (C) is incorrect because a federal question has not been presented, as described above. (D) is incorrect because, even if the plaintiff's claim exceeds $75,000, diversity of citizenship would not exist because the plaintiff and defendant are both citizens of State A.

A homeowner, domiciled in State A, hired a contractor, domiciled in State B, to build an addition to his residence. Dissatisfied with the contractor's work, the homeowner refused to pay the contractor. The contractor promptly commenced a contract action against the homeowner by filing a complaint in the federal court for State B, seeking damages in excess of $75,000. The contractor's lawyer went the homeowner's residence in State A and attempted to hand the summons and complaint to a man who was painting the front porch of the residence. The lawyer told the man that he was serving legal papers. When the man refused to accept the papers, the lawyer touched the papers to the man's arm and placed them on a nearby bench on the porch. The lawyer assumed that the man was the homeowner. In fact, he was the homeowner's neighbor, whom the homeowner had hired to paint the porch while he was vacationing. What is the homeowner's best argument that service was improper? A. Process was served in State A. B. Process was served by the contractor's lawyer. C. The man was the homeowner's neighbor. D. The man refused to accept the papers.

The homeowner's best argument is that the man was a neighbor. Under Rule 4, an individual defendant may be served by leaving a copy of the papers at the individual's dwelling or place of abode with someone of suitable age and discretion who resides there. Here, service on the neighbor was improper because he did not reside there. (A) is incorrect because process can be served in any state. It need not be served in the state where the action was commenced. (B) is incorrect because process may be served by any person who is at least 18 years of age and not a party. Although the lawyer represents a party, he is not himself a party and is likely at least 18 years old. (D) is incorrect because service is valid even if the person served refuses to take the papers, as long as the process server informs him of the nature of the papers and leaves them in his presence. (Even if the process server is unable to touch the papers to the person and leaves them nearby, most courts would hold that service was proper.)

A hotelier entered into a contract with a merchant to have the merchant build and deliver custom furniture having a total cost of $1 million. The hotelier is a resident of State A and the merchant is a resident of State B. The merchant completed the furniture but the hotelier refused delivery, claiming that the furniture was not as the hotelier and merchant had agreed. The hotelier then filed an action against the merchant in federal district court, claiming breach of contract and seeking the return of a $100,000 deposit. The merchant filed an answer, denying liability and denying that it breached the contract. At the same time, the merchant filed and served a counterclaim asserting that the hotelier had breached the contract and seeking the $900,000 balance due. A week after being served with the merchant's answer and counterclaim, the hotelier filed a voluntary dismissal of the action. Is the hotelier's dismissal effective? A. Yes, because the party that commenced an action may always terminate it. B. Yes, because a plaintiff may voluntarily dismiss an action of right if the plaintiff does so within 21 days of the defendant filing an answer. C. No, because, while the hotelier otherwise would have had the right to dismiss the action, the hotelier cannot voluntarily dismiss the action because of the merchant's pending counterclaim. D. No, because, after a defendant files an answer, an action may be dismissed only by a stipulation signed by all parties or by an order of the court.

The hotelier's dismissal is not effective. If the defendant has not answered or filed a motion for summary judgment, the plaintiff may dismiss her case by filing a notice of dismissal. When a voluntary dismissal without leave of court is not available (i.e., there has been an answer, motion, or previous dismissal), the court has discretion to grant dismissal on such terms and conditions as the court deems proper. The dismissal is without prejudice unless the court specifies otherwise. If there is a counterclaim pending in the action, there can be no dismissal over the defendant's objection unless the counterclaim remains pending. (A) is incorrect because, as explained above, there are limitations on when a plaintiff may voluntarily dismiss her claim as of right. (B) is incorrect because the statement regarding the time period is not accurate. The plaintiff may voluntarily dismiss the action before the defendant files an answer. Once the defendant files an answer, the plaintiff's ability to voluntarily dismiss the action as of right is cut off. (C) is incorrect because the plaintiff would still not be able to voluntarily dismiss the action as of right even if the counterclaim did not exist, because the defendant has filed an answer.

An office furniture supplier filed a breach of contract action against a law firm in federal district court to recover the balance due on an account for furniture it supplied. The law firm filed a motion to dismiss the action on the ground that service of process was improper. Following a hearing, the court held that service was proper and sufficient. Two more months passed without the law firm filing an answer. The supplier then filed a motion to have the clerk of court make an entry of default, and the clerk did so. What procedure must the supplier follow to obtain a default judgment against the law firm? A. File a motion to have the clerk of court enter the default judgment, and the clerk may do so without the law firm receiving any further notice of the motion. B. File a motion to have the clerk of court enter the default judgment, and the clerk may do so as long as the law firm receives additional notice of the motion for default judgment. C. File a motion to have the judge enter the default judgment, and the judge may do so without the law firm receiving any further notice of the motion for default judgment. D. File a motion to have the judge enter the default judgment, and the judge may do so as long as the law firm receives additional notice of the motion for default judgment.

The judge may enter the default judgment as long as additional notice is provided to the law firm. A defendant against whom a default is entered loses the right to contest liability. However, the amount of damages must still be determined before a default judgment may be entered, and the defaulting party can be heard at the hearing for damages. If the defendant has "appeared," even though he has not answered, he must be notified of the request for a default judgment by first-class mail at least seven days before the hearing on the application for a default judgment. Appearance includes any actual formal appearance before the court and any other action that clearly indicates that the defendant intends to contest the case on the merits (e.g., the defendant's continued settlement negotiations). Furthermore, an appearance cuts off the clerk's ability to enter a default judgment. (A) is incorrect because the clerk may not enter a default judgment if the defendant has appeared and, because the law firm has appeared, it must be provided with notice. (B) is incorrect because the clerk may enter a default judgment only if the defendant has not appeared. (C) is incorrect because the law firm must be provided with notice of the hearing for a default judgment because it has appeared in the action.

The plaintiff and defendant in a negligence action filed in federal district court were in a two-car accident. The defendant's attorney properly gave notice of and commenced a deposition of an eyewitness to the accident. Attorneys for both parties were present at the deposition. During the deposition, the defendant's attorney asked the witness a leading question, and the plaintiff's attorney did not object. At the time of trial, the eyewitness was out of the country. Therefore, the defendant's attorney sought to read a portion of the witness's deposition testimony into evidence at trial, including the leading question and the witness's response to it. The plaintiff's attorney objected at trial to the reading of the leading question and the witness's response. Is the leading question and response properly admissible in evidence at trial? A. No, because none of the deposition testimony is admissible in evidence under these circumstances. B. No, because, while the deposition testimony is generally admissible under these circumstances, the court may exclude the leading question and answer on the basis of the plaintiff's objection at trial. C. Yes, because the deposition testimony is generally admissible under these circumstances, and the plaintiff waived any objection to the leading question because the plaintiff did not object to the question at the deposition. D. Yes, because once a part of the deposition testimony is admissible under these circumstances, all of the deposition is admissible regardless of the plaintiff's objection.

The leading question and response are properly admissible. Errors of any kind which could have been obviated if promptly presented are waived unless seasonable objection is made at the time of taking the deposition (this applies to form of questions, oath, conduct of parties, etc.). Because the plaintiff's attorney failed to object at the deposition as to the form of the question, he waived the objection at trial. (A) is incorrect. A deposition may be used against any party who was present at the deposition or had notice of it: (i) to impeach the testimony of the deponent as a witness; (ii) for any purpose if the court finds that the deponent (including a party-deponent) is dead, at a distance greater than 100 miles from the place of trial (unless the absence was procured by the party offering the deposition), or unable to testify because of age, sickness, etc.; or (iii) for any purpose if the deponent is an adverse party. Because the plaintiff's attorney was at the deposition, and because the deponent here is out of the country and no facts indicate that the defendant or his attorney procured the absence, the deposition may be used at trial. (B) is incorrect for the reason stated above-by failing to object to the leading question, the plaintiff's attorney waived the objection. (D) is incorrect. It is true that, if a party offers in evidence only part of a deposition, an adverse party may require the offeror to introduce other parts that in fairness should be considered with the part introduced, and any party may itself introduce any other parts. However, any objections made during the taking of the deposition must still be preserved. The key fact here is that the plaintiff's attorney failed to object.

A patient sued a surgeon for medical malpractice, alleging that the surgeon used an improper procedure during cardiac surgery, thereby permanently injuring the patient. The surgeon then wrote to a colleague, who was a renowned cardiologist, asking whether in her opinion the procedure was proper. In response, the colleague stated that the procedure was likely improper, but that she was unsure whether the patient's injuries resulted from it. After receiving the colleague's letter, the surgeon did not have any further communication with her about the matter. Is the letter discoverable? A. No, because the colleague did not have first-hand knowledge regarding the surgery that she was evaluating. B. No, because the colleague will not testify at trial on behalf of the doctor. C. Yes, because the colleague was not retained or specially employed by the doctor. D. Yes, because the colleague is not an expert.

The letter is not discoverable because the colleague is a nontestifying expert. An expert is a person with knowledge, skill, experience, training, or education in a particular field. The opinion of an expert consulted in anticipation of litigation is discoverable if a party intends to call the expert as a trial witness. Conversely, the opinion of an expert who is retained in anticipation of litigation but who is not expected to testify at trial (i.e., a consulting expert) is discoverable only upon a showing of exceptional circumstances under which it is impracticable to obtain facts or opinions by other means. Absent such circumstances, these opinions are not discoverable, regardless of whether the nontestifying expert was informally consulted or whether she was retained or specially employed. Thus, the doctor need not produce the letter from his colleague, an expert in cardiology, because the doctor does not expect to call her as a trial witness. (A) is incorrect because an expert need not have first-hand knowledge of facts to give an opinion based on those facts. In any event, although a fact witness's lack of knowledge may in some circumstances provide a reason for excluding testimony at trial, it is not a valid ground for resisting discovery of relevant information. (C) is incorrect because the opinion of a nontestifying expert is nondiscoverable regardless of whether she is formally retained or not. (D) is incorrect because the cardiologist is an expert. The question states that she is a renowned cardiologist and that the doctor is consulting her about a cardiac surgery.

A plaintiff, a citizen of State A, sued a defendant, a citizen of State B, alleging that the defendant violated the Civil Rights Act by refusing to serve the plaintiff in her restaurant. The plaintiff brought his suit in state court in State B, asking for damages of $100,000. The defendant seeks to remove the case to the United States District Court for the District of State B, and the plaintiff opposes removal. Is the case properly removable? A. No, because the defendant is a citizen of the forum state. B. No, because the state court has concurrent jurisdiction, making removal improper. C. Yes, because diversity of citizenship exists and the amount in controversy is more than $75,000. D. Yes, because a federal question has been presented.

The motion should be granted. The case may be removed because the federal court has federal question jurisdiction over the plaintiff's action, because it is based on the Civil Rights Act. (A) is incorrect. The limitation on removal by defendants who are citizens of the state in whose court the action was brought applies only to diversity actions. (C) is incorrect. Although diversity of citizenship appears to be present in this action, it is only incidental. The removal statute provides that "any civil action of which the district courts have [federal question jurisdiction] shall be removable without regard to the citizenship or residence of the parties." [28 U.S.C. §1441(b)] When both a federal question and diversity jurisdiction exist, the federal question jurisdiction normally "trumps" diversity jurisdiction. In any event, if this case were to be solely based on diversity, the defendant could not remove because she is a citizen of the forum state. (B) is incorrect because, although a state court may have concurrent jurisdiction, this jurisdiction, by itself, would not prevent removal.

A pilot who owned an air charter service adjacent to a crop duster's hangar at a municipal airport sued the crop duster in federal court for allowing chemicals to contaminate the ground outside the pilot's office in violation of federal standards. The pilot sought a preliminary injunction against further operations by the crop duster. The court denied the injunction, but agreed to hear the case on the merits. The pilot believes that the pesticides pose an immediate danger, so he appeals the denial of the injunction. May the pilot immediately appeal the denial of the injunction? A. No, because the order is not final. B. No, because the trial court agreed to hear the case on the merits. C. Yes, because federal law permits appeal of orders denying an injunction. D. Yes, because the pilot will suffer immediate harm.

The pilot may immediately appeal the court's decision, because federal law permits appeal of orders denying an injunction. Interlocutory orders denying an injunction may be appealed in the federal court system. [28 U.S.C. §1292(a)] This constitutes an exception to the general rule that interlocutory orders are not appealable because they are not final. Thus, (A) and (B) are incorrect. (D) is incorrect. Although the pilot may suffer immediate harm, it does not provide a basis for an immediate appeal.

The owner of property bordering a river properly brought suit in federal court against a power company, alleging that the power company dumped radioactive waste water from one of its nuclear plants into the river, causing the property owner many health issues and resulting in the diminution of his property's value. The suit sought damages and a preliminary injunction forbidding the power company from dumping any waste into the river. The power company was provided with notice of the hearing, to which it sent its lawyer. During the hearing, the power company claimed that it has complied with applicable state and federal environmental regulations, and stated that it will continue to comply with the regulations if the injunction is denied. The judge granted the preliminary injunction and set a date for a pretrial conference, as required by the Federal Rules of Civil Procedure. May the power company immediately appeal the judge's granting of the preliminary injunction? A. No, because the order is not final. B. No, because the question has not been certified as one falling within the Interlocutory Appeals Act. C. Yes, but only if the power company takes a default judgment, thereby making the judgment final. D. Yes, because the order is immediately appealable as of right.

The power company may immediately appeal. Although the general rule is that only a final order may be appealed, there are certain exceptions to that rule. A party may appeal, as of right, any order granting, continuing, modifying, refusing, dissolving, or refusing to dissolve or modify an injunction. (A) is incorrect because the granting, continuing, modifying, etc. of an injunction is an exception to the final order rule, as explained above. (B) is incorrect because there is no need for the power company to have the question certified under the Interlocutory Appeals Act. Review under the Interlocutory Appeals Act is discretionary and may be available when: (i) the trial judge certifies that the interlocutory order involves a controlling question of law, as to which there is substantial ground for difference of opinion, and immediate appeal from the order may materially advance the ultimate termination of the litigation; and (ii) the court of appeals then agrees to allow the appeal. The process is not required for orders involving an injunction. By rule, an order involving an injunction is appealable as of right, as explained above. (C) is incorrect because a final judgment is not required before a party may appeal the granting of an injunction. Additionally, taking a default judgment is a risky strategy, as it precludes defending the case on the merits if the party loses on appeal.

A customer choked on a bone in her fish fillet while eating at a restaurant. The restaurant's manager, who was not in the restaurant when the incident occurred, met with the customer and the restaurant employees who witnessed the incident. As required by state law, the manager prepared a report regarding the incident and filed it with the state board of health. The manager then accepted a job in another country and moved there. The customer subsequently commenced a civil action against the restaurant in a federal court, alleging negligence and a breach of the warranty of merchantability. The customer's attorney served a request for all documents and reports prepared by the restaurant relating to the incident. Must the restaurant produce the manager's report? A. Yes, because the manager prepared the report to comply with state law rather than to prepare for trial. B. Yes, because the customer can establish that she has a substantial need for the report to prepare her case. C. No, because the manager's report is hearsay and therefore not discoverable. D. No, because the report constitutes work product and is therefore not discoverable.

The restaurant must produce the report because it is relevant, proportional to the needs of the case, and it was not prepared in anticipation of litigation. Generally, a party may obtain discovery regarding any matter that is proportional to the needs of the case and that is relevant to any party's claim or defense, provided the matter is not subject to a privilege or to the exception for trial preparation materials, also called work product. Trial preparation materials are documents or other tangible things prepared in anticipation of litigation or for trial. Here, the manager's report is relevant because it contains facts relating to the incident that is the basis of plaintiff's claim. Moreover, it is not subject to the exception for trial preparation materials because it was prepared in the ordinary course of business to comply with state law, not in anticipation of litigation. Finally, the costs of producing the report likely would not be prohibitive. (B) is incorrect because it invokes the exception to the rule against discovery of work product, namely that work product is discoverable if the party seeking it "has substantial need for the materials to prepare its case and cannot, without undue hardship, obtain their substantial equivalent by other means." This exception is not pertinent because the manager's report is not work product. (C) is incorrect because material need not be admissible as evidence to be discoverable. The standard is relevancy, not admissibility. (D) is incorrect because, as discussed above, the report is not work product.

A former shareholder of a corporation filed an action against the corporation's board of directors, alleging that the directors' wrongful actions diminished the value of the corporation and its stock. The former shareholder served a request for production of documents that included a request for any documents relating to the value of the corporation. The corporation produced a number of documents, but withheld certain documents that were written by its attorneys, claiming that the documents were protected from discovery under the attorney-client privilege. Without contacting or conferring with the corporation or its lawyer, the former shareholder filed a motion to compel production of the withheld documents, claiming that they were not covered by the privilege. The court ruled that the documents were not privileged and had to be produced. What will be the likely result if the former shareholder seeks to recover costs or attorneys' fees related to the motion or to have sanctions imposed on the corporation? A. The shareholder may not recover costs or attorneys' fees or have sanctions imposed. B. The shareholder may recover the costs of asserting the motion to compel but may not recover any attorneys' fees or have sanctions imposed. C. The shareholder may recover the costs and attorneys' fees incurred in asserting the motion to compel, but may not have sanctions imposed. D. The shareholder may recover costs and attorneys' fees incurred in asserting the motion to compel, and may have sanctions imposed.

The shareholder is not likely to recover costs or attorneys' fees or have sanctions imposed. If a party fails to provide discovery or provides incomplete discovery, including disclosures and answers to interrogatories and deposition questions, the other party may move to compel discovery. However, a motion to compel must certify that the moving party has made a good faith attempt to confer with the opponent to obtain the discovery without court intervention. The certification (and an actual attempt at resolving the discovery dispute without court intervention) is a prerequisite to an award of reasonable expenses (which includes attorneys' fees). Here, the movant cannot certify in a motion to compel that he made a good faith effort to obtain discovery without court action because the facts indicate that he made no such effort. (B) is incorrect. The applicable federal rule [Federal Rule 37] does not distinguish between costs and attorneys' fees. (C) is incorrect. As explained above, an attempt to confer with the opponent (and the certification that the attempt was made) is a prerequisite to an award of reasonable costs. On the other hand, (C) correctly states that sanctions are not available. Sanctions generally may be imposed only for violating a court order on discovery. (D) is incorrect because, as explained above, an attempt to confer and certification are required before an award of reasonable costs may be made, and sanctions generally may be imposed only for violating a court order on discovery.

A shopper sued a grocery store in federal court for a back injury after a fall in the store. The shopper provided the store with a medical report from his own doctor. However, the store located a physician specializing in back injuries who agreed to examine the shopper independently. The store sent a discovery notice to the shopper instructing him to call the physician to set up an appointment. The shopper declined, asserting that he would not do so without a court order. The store responded that if the shopper insisted on a court order, it will also ask the court to impose sanctions on the shopper for failing to cooperate in discovery. Is the store's position as to sanctions justifiable? A. Yes, because the shopper must cooperate with the store during the discovery period. B. Yes, if the physician is deemed to be a neutral physician. C. No, if the shopper's original doctor was also a back specialist. D. No, because the shopper need not submit to an examination by a physician without a court order.

The store's position as to sanctions is not justifiable. Although the shopper's physical condition is in issue, he need not submit to an exam by another physician without a court order obtained on a showing of good cause. The shopper's insistence on a court order thus does not constitute a failure to cooperate in discovery. [Fed. R. Civ. P. 35] For this reason, (A) is incorrect. (B) is incorrect because a court order is still required. Furthermore, there is no prohibition against the physician being employed by the requesting party. (C) is incorrect because a party may not avoid the examination by claiming that his prior examination was sufficient.

A shopkeeper sued a supplier in federal court in a dispute over incomplete and incorrect deliveries. The shopkeeper and supplier presented their evidence at trial, and the case went to the jury with no motions being made at trial. The jury found in favor of the shopkeeper, and judgment was entered accordingly. Believing that there was insufficient evidence for a reasonable jury to find in favor of the shopkeeper, the supplier wants to overturn the verdict. Which motion should the supplier file? A. A motion for summary judgment. B. A motion for judgment as a matter of law. C. A renewed motion for judgment as a matter of law. D. A motion for a new trial.

The supplier should file a motion for a new trial. A party can file a motion for a new trial no later than 28 days after judgment was entered. The supplier is within the time limit and can therefore file. (A) is incorrect because a motion for summary judgment should be filed before trial begins. (B) is incorrect because a motion for judgment as a matter of law should be made before submission of the case to the jury. (C) is incorrect because a renewed motion for judgment as a matter of law must be filed no later than 28 days after entry of judgment, and the party making the motion must have moved for judgment as a matter of law at some time during the trial. Here, the supplier did not move for judgment as a matter of law during the trial, and as a result the renewed motion is not available to him.

A taxi crashed into a telephone pole. The taxi's passenger commenced a negligence action against the taxi driver, properly serving the taxi driver with the summons and complaint. The taxi driver, being an independent contractor with no right to indemnity from the taxi's owner, turned the summons and complaint over to his insurance company. After trial, the jury returned a verdict in favor of the passenger, finding that the taxi driver's negligence was the sole cause of the crash. The passenger then commenced a second action, this time against the owner of the taxi. The owner also had no right of indemnity from the taxi driver. The passenger alleged that the taxi driver's negligence was the sole cause of the crash and that the owner was vicariously liable. The passenger immediately moved for partial summary judgment against the owner based on the finding of the taxi driver's negligence in the prior action. If the court denies the motion, what is the most likely reason? A. Claim preclusion bars the passenger's claim against the owner. B. The taxi owner is not precluded from litigating the issue of whether the taxi driver was negligent. C. The passenger waived her right to sue the owner by not joining the owner as a defendant in the first action. D. The taxi driver is a necessary and indispensable party to the second action.

The taxi owner is not precluded from litigating that issue. For a party to be bound by issue preclusion, (i) there must have been a final judgment; (ii) the issue must have been actually litigated and determined; (iii) the issue must have been essential to the judgment; and (iv) the party to be bound by the prior judgment must have been a party to the prior action or in privity with a party to the prior action. Here, it is the last element that is missing. The taxi owner was not a party to the prior action, and there is no indication that the taxi owner was in privity with the taxi driver. (A) is incorrect for a similar reason. Claim preclusion applies when the earlier case and the later case are brought by the same claimant against the same defendant (or where a party in the later case is in privity with a party in the earlier case). Here, the claim is against a different defendant who was not in privity with the previous defendant. (C) and (D) are incorrect because there generally is no requirement to join joint tortfeasors in a single action. (The exception being if state law requires it.) The risk the plaintiff runs in such a situation is that she loses the first case and thereafter might be bound by issue preclusion in subsequent actions.

To receive a jury trial, a party generally must: A. File a written demand and serve it on all the parties within 28 days after the service of the last pleading directed to the jury-triable issue B. File a written demand and serve it on any opposing parties within 14 days after the service of the last pleading directed to the jury-triable issue C. File a written demand and serve it on all the parties within 14 days after the service of the last pleading directed to the jury-triable issue D. File a written demand and serve it on any opposing parties within 28 days after the service of the last pleading directed to the jury-triable fact issue

Under Rule 38, to receive a jury trial a party must file a written demand and serve it on all the parties within 14 days after the service of the last pleading directed to the jury-triable issue. Otherwise, the right to a jury trial generally is deemed waived. Note, however, that the Supreme Court has held that, in the absence of compelling reasons to the contrary, a court should grant relief from the waiver if the issue is normally tried by a jury. Filing a written demand within 28 days after the service of the last pleading directed to the jury-triable issue may be too late. The applicable period is 14 days. Filing a written demand on any opposing parties may be insufficient where multiple parties are involved. All parties must be served.

Under the federal discovery rules, the parties must confer to consider their claims and defenses, the possibility of settlement, initial disclosures, any issues concerning the preservation of evidence, and a discovery plan. Unless a different time is set by court order or stipulation, within how many days after this meeting must the parties provide the names, addresses, and telephone numbers of individuals likely to have discoverable information that the disclosing party may use to support its claims or defenses? A. Within 28 days after the meeting B. Within 14 days after the meeting C. Within 10 days after the meeting

Unless a different time is set by court order or stipulation, within 14 days after the meeting the parties must provide the names, addresses, and telephone numbers of individuals likely to have discoverable information that the disclosing party may use to support its claims or defenses (unless needed solely for impeachment). Other disclosures that must be made at this time include: (1) copies or descriptions of documents, electronically stored information, and tangible things that are in the disclosing party's possession or control and that the disclosing party may use to support its claims or defenses, unless solely for impeachment, (2) computation of damages claimed by the disclosing party and copies of materials upon which the computation is based; and (3) copies of insurance agreements under which an insurer might be liable for all or part of any judgment that might be entered. Within 28 days after the meeting and within 10 days after the meeting are both incorrect.

Can either subject matter jurisdiction or venue be conferred by agreement? A. Yes as to both subject matter jurisdiction and venue B. Yes as to subject matter jurisdiction C. Yes as to venue D. No as to both subject matter jurisdiction and venue

Venue can be conferred by agreement, but subject matter jurisdiction cannot be conferred by agreement. Subject matter jurisdiction and venue are very often confused. Subject matter jurisdiction is the power of the court to adjudicate the matter before it, whereas venue relates to the proper geographic district in which to bring the action.

Which of the following statements is correct? A. Objection cannot be made to improper venue B. Venue is considered to be objected to unless expressly waived C. Venue objections cannot be waived D. Venue is considered to be waived unless a timely objection is made

Venue is considered to be waived unless a timely objection (in a pre-pleading motion or, where no such motion is made, in the answer) is made to the improper venue. There is no presumed objection to venue. However, unlike jurisdiction over the subject matter, venue may be waived by the parties.

An automobile crash victim intends to file a negligence action based on diversity of citizenship jurisdiction against a driver in federal court for injuries sustained in an automobile accident that occurred in State A. The victim resides in State B, and the driver resides in State C. State A, State B, and State C all have only a single federal judicial district. In which federal district court(s) is venue proper? A. The District of State A only. B. The District of State C only. C. The District of State A and the District of State C. D. The District of State A, the District of State B, and the District of State C.

Venue is proper in the District of State A and the District of State C. Federal venue in civil actions is proper in (i) the district where any defendant resides, if all defendants are residents of the state in which the district is located; and (ii) the district in which a substantial part of the events or omissions giving rise to the claim occurred. If there is no district anywhere in the United States that satisfies (i) or (ii), the action may be brought in a judicial district in which any defendant is subject to the court's personal jurisdiction with respect to such action. Here, the defendant resides in the District of State C, and the accident occurred in the District of State A, making those two venues proper under (i) and (ii), which is choice (C). Hence, (A), (B), and (D) are incorrect.

When he turned 21, a young man who lived in State A decided to move to a city in State B. He loaded all his possessions in his truck and trailer and set out for State B. While en route, he was involved in a serious accident in State C with a woman driving an SUV, injuring both parties. Because of his injuries, the man has remained in State C for several months. However, he still intends to relocate to the city in State B as soon as he has recuperated and is able to travel. The woman, a citizen and resident of State D, is preparing to file a negligence action in federal district court against the man for the injuries she suffered in the State C accident. If the woman files the action before the man proceeds to State B, in what federal district or districts is venue proper? A. The District of State C only. B. The District of State C and the District of State A. C. The District of State C and the District of State B. D. The District of State C, the District of State A, and the District of State B.

Venue is proper in the District of State C and the District of State A. Federal venue in civil actions is proper in (1) the district where any defendant resides, if all defendants are residents of the state in which the district is located; and (2) the district in which a substantial part of the events or omissions giving rise to the claim occurred. If there is no district anywhere in the United States that satisfies (1) or (2), the action may be brought in a judicial district in which any defendant is subject to the court's personal jurisdiction with respect to such action. Here, the District of State C is a proper venue because a substantial part of the events giving rise to the claim occurred there. Additionally, at the time of the accident, the man's domicile was in State A. It remains his domicile until he acquires a new one by being physically present in a new place while intending to make that new place his permanent home. The man did not acquire a new domicile in State B because he has yet to be physically present there (although he does intend to make it his permanent home). As a result, (B) is the correct answer. (A) is not correct because the answer does not discuss the possibility of venue being based on the man's domicile. State C was not his domicile, because he does not intend to live indefinitely in State C. As stated above, State A remains his domicile. (C) and (D) are not correct because, as discussed above, the man has not made either place his domicile.

A zoologist was moving to State A from State B, where she had lived her entire life. She was driving a truck and trailer that contained all of her possessions across the country when she was involved in a severe accident in State C. After being released from the hospital, but before suit was filed, the zoologist proceeded to State A, where she rented an apartment and found a job. The driver of an automobile involved in the severe accident intends to file a negligence action against the zoologist in federal district court. The driver is a citizen and resident of State D. In which federal district(s) is venue proper? A. The District of State C only. B. The District of State C and the District of State B. C. The District of State C and the District of State A only. D. The District of State C, the District of State D, and the District of State A.

Venue is proper in the District of State C and the District of State A. Federal venue in civil actions is proper in (i) the district where any defendant resides, if all defendants are residents of the state in which the district is located; and (ii) the district in which a substantial part of the events or omissions giving rise to the claim occurred. If there is no district anywhere in the United States that satisfies (i) or (ii), the action may be brought in a judicial district in which any defendant is subject to the court's personal jurisdiction with respect to such action. Thus, the District of State C is a proper venue because a substantial part of the events giving rise to the claim occurred there, and the District of State A is a proper venue because that is where the zoologist/defendant was domiciled at the time the case was filed. Accordingly, (A) and (B) are incorrect. (D) is incorrect because, in federal court, venue cannot be based on the plaintiff's residence.

When an action involves multiple claims or parties, and a judgment is entered that disposes of only some of the parties or claims, the judgment: A. Generally is immediately appealable regarding the parties who were disposed B. Is not immediately appealable until a final judgment is rendered as to all parties and all claims C. Is immediately appealable only to the extent that the court makes an express determination that there is no just reason for delay D. Generally is immediately appealable as to the claims that were disposed

When an action involves multiple claims or parties, and the court enters a judgment as to fewer than all the claims or all the parties, it is deemed a final, appealable judgment only to the extent the court makes an express determination that there is no just reason for delay. Generally, a judgment as to only some of the claims or parties is not immediately appealable, regarding the parties who were disposed or as to the claims that were disposed. It is untrue that a judgment regarding only some of the parties or claims is not immediately appealable until a final judgment is rendered as to all claims and all parties because such judgments are appealable if the judge expressly makes a determination that there is no just reason for delay.


Ensembles d'études connexes

DOSAGE EXAM PRACTICE 30 Qs 1.5 HOURs

View Set

AFJROTC // Chapter 6 Lesson 2 Preparing Your Résumé Test

View Set

Fundamental Chapter 37: Urinary Elimination

View Set

Anatomy and Physiology I Final Exam

View Set

HESI Module 1 Exam-Developmental Stages and Transitions

View Set

QA interview Questions, **QA-Interview**

View Set